You are on page 1of 123

www.jntuworld.

com

www.jwjobs.net

Electronic Measurements & Instrumentation

Question & Answers

UNIT-1

1. What are the basic performance characteristics of a system? Ans:


STATIC CHARACTE RISTICS The static characteristics of an instrument are, in general, considered for instruments which are used to measure an unvarying process condition. All the static performance characteristics are obtained by one form or another of a process called calibration. There are a number of related definitions (or characteristics), which are described below, such as accuracy% precision, repeatability, resolution, errors, sensitivity, etc. l. Instrument: A device or mechanism used to determine the present value of the quantity under measurement. 2. Measurement: The process of determining the amount, degree, or capacity by comparison (direct or indirect) with the accepted standards of the system units being used. 3. Accuracy: The degree of exactness (closeness) of a measurement compared to the expected (desired) value. 4. Resolution: The smallest change in a measured variable to which an instrument will respond. 5. Precision: A measure of the consistency or repeatability of measurements, i.e. successive readings does not differ. (Precision is the consistency of the instrument output for a given value of input). 6. Expected value: The design value, i.e. the most probable value that calculations indicate one should expect to measure. 7 Error: The deviation of the true value from the desired value. 8. Sensitivity: The ratio of the change in output (response) of the instrument to a change of input or measured variable.

DYNAMIC CHARACTERISTICS Instruments rarely respond instantaneously to changes in the measured variables. Instead, they exhibit slowness or sluggishness due to such things as mass, thermal capacitance, fluid capacitance or electric capacitance. In addition to this, pure delay in time is often encountered where the instrument waits for GRIET/ECE

www.jntuworld.com

www.jntuworld.com

www.jwjobs.net

Electronic Measurements & Instrumentation

Question & Answers

some reaction to take place. Such industrial instruments are nearly always used for measuring quantities that fluctuate with time. Therefore, the dynamic and transient behavior of the instrument is as important as the static behavior. The dynamic behavior of an instrument is determined by subjecting its primary element (sensing element) to some unknown and predetermined variations in the measured quantity. The three most common variations in the measured quantity are as follows: l. Step change in which the primary element is subjected to an instantaneous and finite change in measured variable. 2. Linear change, in which the primary element is following a measured variable, changing linearly with time. 3, Sinusoidal change, in which the primary element follows a measured variable, the magnitude of which changes in accordance with a sinusoidal function of constant amplitude . The dynamic characteristics of an instrument are (i) speed of response, (ii) Fidelity, (iii) lag, and (iv) dynamic error. (i) Speed of Response: It is the rapidity with which an instrument responds to changes in the measured quantity. (ii) Fidelity: It is the degree to which an instrument indicates the changes in the measured variable without dynamic error (faithful reproduction). (iii) Lag: It is the retardation or delay in the response of an instrument to changes in the measured variable. (iv) Dynamic Error: It is the difference between the true values of a quantity changing with time and the value indicated by the instrument, if no static error is assumed. When measurement problems are concerned with rapidly varying quantities, the dynamic relations between the instruments input and output are generally Defined by the use of differential equations

2. What are the different types of static errors in a system? Ans:


The static error of a measuring instrument is the numerical difference between the true value of a quantity and its value as obtained by measurement, i.e. repeated measurement of the same quantity give different indications. Static errors are categorized as gross errors or human errors, systematic errors and Random errors. 1. Gross Errors GRIET/ECE

www.jntuworld.com

www.jntuworld.com

www.jwjobs.net

Electronic Measurements & Instrumentation

Question & Answers

This error is mainly due to human mistakes in reading or in using instruments or errors in recording observations. Errors may also occur due to incorrect adjustments of instruments and computational mistakes. These errors cannot be treated mathematically. The complete elimination of gross errors is not possible, but one can minimize them .Some errors are easily detected while others may be elusive. One of the basic gross errors that occur frequently is the improper use of an Instrument the error can be minimized by taking proper care in reading and recording the measurement parameter. In general, indicating instruments change ambient conditions to some extent when connected into a complete circuit. 2. Systematic Errors These errors occur due to shortcomings of, the instrument, such as defective or worn parts, or ageing or effects of the environment on the instrument. These errors are sometimes referred to as bias, and they influence all measurements of a quantity alike. A constant uniform deviation of the operation of an instrument is known as a systematic error. There are basically three types of systematic errors (i) Instrumental, (ii) Environmental, and (iii) Observational (i) Instrumental Errors Instrumental errors are inherent in measuring instruments, because of their mechanical structure. For example, in the D'Arsonval movement friction in the bearings of various moving components, irregular spring tensions, stretching of the spring or reduction in tension due to improper handling or over loading of the instrument. Instrumental errors can be avoided by (a) Selecting a suitable instrument for the particular measurement applications. (b) Applying correction factors after determining the amount of instrumental error. (c) Calibrating the instrument against a standard. (ii) Environmental Errors Environmental errors are due to conditions external to the measuring device, including conditions in the area surrounding the instrument, such as the effects of change in temperature, humidity, barometric pressure or of magnetic or electrostatic fields. These errors can also be avoided by (i) air conditioning, (ii) hermetically sealing certain components in the instruments, and (iii) using magnetic shields. (iii) Observational Errors Observational errors are errors introduced by the observer. The most common error is the parallax error introduced in reading a meter scale, and the error of estimation when obtaining a reading from a meter scale.

GRIET/ECE

www.jntuworld.com

www.jntuworld.com

www.jwjobs.net

Electronic Measurements & Instrumentation

Question & Answers

These errors are caused by the habits of individual observers. For example, an observer may always introduce an error by consistently holding his head too far to the left while reading a needle and scale reading. In general, systematic errors can also be subdivided into static and dynamic Errors. Static errors are caused by limitations of the measuring device or the physical laws governing its behavior. Dynamic errors are caused by the instrument not responding fast enough to follow the changes in a measured variable.

3. What is the method used to calculate the errors in an instrument? Ans:


ERROR IN MEASUREMENT Measurement is the process of comparing an unknown quantity with an accepted standard quantity. It involves connecting a measuring instrument into the system under consideration and observing the resulting response on the instrument. The measurement thus obtained is a quantitative measure of the so-called "true value" (since it is very difficult to define the true value, the term "expected value" is used). Any measurement is affected by many variables; therefore the results rarely reflect the expected value. For example, connecting a measuring instrument into the circuit under consideration always disturbs (changes) the circuit, causing the measurement to differ from the expected value. Some factors that affect the measurements are related to the measuring instruments themselves. Other factors are related to the person using the instrument. The degree to which a measurement nears the expected value is expressed in terms of the error of measurement. Error may be expressed either as absolute or as percentage of error. Absolute error may be defined as the difference between the expected value of the variable and the measured value of the variable, or e= Where e=absolute errors; Yn=expected value; Xn=measured value; Therefore %error = (absolute value/expected value )*100=(e/Yn)*100 Y n- X n

Therefore %error= It is more frequently expressed as an accuracy rather than error. Therefore A=1Where A is the relative accuracy Accuracy is expressed as % accuracy GRIET/ECE

www.jntuworld.com

www.jntuworld.com

www.jwjobs.net

Electronic Measurements & Instrumentation

Question & Answers

a=100%-%error a=A*100% (where a=%accuracy)

4. Describe the function of the DC-Voltmeter and multi range voltmeter and explain their operation? Ans: DC-Voltmeter
A basic D'Arsonval movement can be converted into a dc voltmeter by adding a series resistor known as multiplier, as shown in the figure. The function of the multiplier is to limit the current through the movement so that the current does not exceed the full scale deflection value. A dc voltmeter measures the potential difference between two points in a dc circuit or a circuit component. To measure the potential difference between two points in a dc circuit or a circuit component, a dc voltmeter is always connected across them with the proper polarity. The value of the multiplier required is calculated as follows. Im: full scale deflection current of the movement Rm : internal resistance of movement Rs : Multiplier resistance V: full range voltage of the instrument From the circuit of Fig. 4.1 V= Im *( Rm+ Rs) Rs = therefore Rs = = -

The multiplier limits the current through the movement, so as to not exceed the value of the full scale deflection Ifsd. The above equation is also used to further extend the range in DC voltmeter'.

Multi range Voltmeter: As in the case of an ammeter, to obtain a multi range ammeter, a number of shunts are connected GRIET/ECE

www.jntuworld.com

www.jntuworld.com

www.jwjobs.net

Electronic Measurements & Instrumentation

Question & Answers

across the movement with Similarly, a dc voltmeter multi range voltmeter by resistors (multipliers) to provide a greater ranges. The below Figure voltmeter using a three multipliers R1, R2, and V1, V2, and V3. Fig 4.2 multipliers connected in more practical multiplier resistors of a this arrangement, the multipliers are connected in a series string, and the appropriate amount of resistance required in series with the movement.

a multi-position switch. can be converted into a connecting a number of along with a range switch number of workable shows a multi range position switch and three R3, for voltage values can be further modified to series string, which is a arrangement of the multi range voltmeter. In range selector selects the

This arrangement is advantageous compared to the previous one, because all multi1llier resistances except the first have the standard resistance value and are also easily available in precision tolerances. The first resistor or low range multiplier, R4, is the only special resistor which has to be specially manufactured to meet the circuit requirements.

5. Explain the working of solid state voltmeter?


Ans: The below figure shows the circuit of an electronic voltmeter using an IC Op Amp 741C.This is a directly coupled very high gain amplifier. The gain of GRIET/ECE

www.jntuworld.com

www.jntuworld.com

www.jwjobs.net

Electronic Measurements & Instrumentation

Question & Answers

the Op Amp can be adjusted to any suitable lower value by providing appropriate resistance between its output terminal, Pin No. 6, and inverting input, Pin No. 2, to provide a negative feedback. The ratio R2 /R1 determines the gain, i.e. 101 in this case, provided by the Op Amp. The 0.1 pF capacitor across the 100 k resistance R is for stability under stray pick-ups Terminals 1 and 5 are called offset null terminals. A 10 k potentiometer is connected between these two offset null terminals with its centre tap connected to a - 5V supply. This potentiometer is called zero set and is used for adjusting zero output for zero input conditions. The two diodes used are for IC protection. Under normal conditions, they are non-conducting, as the maximum voltage across them is l0 mV. If an excessive voltage, say more than 100 mV appears across them, then depending upon the polarity of the voltage, one of the diodes conducts and protects the IC. A A scale of 50 - 1000 A full scale deflection can be used as an indicator. Ro is adjusted to get maximum full scale deflection.

6. Draw the block diagram of the measuring system and explain the function of each stage of this system? Ans:
The generalized measuring system consists of three main functional elements. They are, 1. Primary sensing element, which senses the quantity under measurement. 2. Variable conversion element, which modifies suitably the output of the primary sensing element 3. Data presentation element that renders the indication on a calibrated scale.

1. Primary Sensing Element The measurement first comes into contact with primary sensing element where the conversion takes place. This is done by a transducer which converts the measurement (or) measured quantity into a usable electrical output. The transduction may be from mechanical, electrical (or) optical to any related form. GRIET/ECE

www.jntuworld.com

www.jntuworld.com

www.jwjobs.net

Electronic Measurements & Instrumentation

Question & Answers

2. Variable Conversion Element The output of the primary sensing element is in the electrical form suitable for control, recording and display. For, the instrument to perform the desired function, it may be necessary to convert this output to some other suitable for preserving the original information. This function is performed by the variable conversion element. A system may require one (or) more variable conversion suitable to it. (a) Variable Manipulation Element The signal gets manipulated here preserving the original nature of it. For example, an amplifier accepts a small voltage signal as input and produces a voltage, of greater magnitude. The output is the same voltage but of higher value, acting as a voltage amplifier. Here the voltage amplifier acts as a variable manipulation element since it amplifies the voltage. The element that follows the primary sensing element in a measurement system is called signal conditioning element. Here the variable conversion element and variable manipulation element are collectively called as Data conditioning element (or) signal conditioning element. (b) Data Transmission Element The transmission of data from one another is done by the data transmission element. In case of spacecrafts, the control signals are sent from the control stations by using radio signals. The stage that follows the signal conditioning element and data transmission element collectively is called the intermediate stage. (c).Data Presentation Element The display (or) readout devices which display the required information about the measurement, forms the data presentation element. Here the information of the measurand has to be conveyed for, monitoring, Control (or) analysis purposes. (a). 1t case of data to be monitored, visual display devices are needed like ammeters; voltmeters and so on are used. (b)In case of data to be recorded, recorders like magnetic tapes, T.V equipment, and storage type C.R T, printers and so on are used.

7. Explain the types of test signals used in determining dynamic measurements applied to a system. Ans:
GRIET/ECE

characteristics of

www.jntuworld.com

www.jntuworld.com

www.jwjobs.net

Electronic Measurements & Instrumentation

Question & Answers

The a dynamic characteristic (or) analysis is classified with respect to time and frequency as time domain analysis and frequency domain analysis (a).In time domain analysis the i/p is applied to the system and the behavior of the system is studied as a function of time. (b) In frequency, domain analysis the i/p is a sinusoidal one and the behavior of the system is studied as a function of frequency. The standard test signals used for time domain analysis are as follows. (i) Step input (ii) Ramp input (iii) Parabolic input (iv) Impulse input. (i) Step Input The continuous time step input u (t) is defined as the discrete time step input a[n] is defined as, U (t) = and discrete time step input u[n] is defined as, u (n) =

Therefore, a unit step input represents a signal which changes its level from 0 to I in zero time and. it reveals a great deal about how quick, the system responds to an abrupt change in the input signal (ii) Ramp Input The ramp input is defined in continuous time as

GRIET/ECE

www.jntuworld.com

www.jntuworld.com

www.jwjobs.net

Electronic Measurements & Instrumentation

Question & Answers

r (t) = and r[n] =

(iii) Parabolic Input


The parabolic input is defined as, r (t) = and the discrete time is defined as, r[n] = The signal are given below,

GRIET/ECE

10

www.jntuworld.com

www.jntuworld.com

www.jwjobs.net

Electronic Measurements & Instrumentation

Question & Answers

This signal is also called as acceleration input since the input signal is proportional to represents a constant acceleration.
(iv) Impulse Input

It is also called as a (delta) function. The continuous time impulse input is given by, square of time and (t) =0 for t0 And discrete time impulse input is given by, (n) =

The unit impulse is defined as the signal which has a zero value everywhere except at t=0.where the magnitude is finite.

GRIET/ECE

11

www.jntuworld.com

www.jntuworld.com

www.jwjobs.net

Electronic Measurements & Instrumentation

Question & Answers

In frequency domain analysis, the system behavior is studied through the sinusoidal signal because the time varying signals such as step, ramp, and parabolic inputs can be expressed in terms of sinusoidal signal of differential amplitudes and frequencies. A continuous time sinusoidal signal is given as Where X(t)=A sin(t + ) A= amplitude = frequency in radians/sec. = phase angle in radians. A sinusoidal signal is an example of a periodic signal, the period of which is T= The discrete time version of a sinusoidal signal is given by, X[n] =A sin (n + ) Where, = angular frequency in radians/cycle.

GRIET/ECE

12

www.jntuworld.com

www.jntuworld.com

www.jwjobs.net

Electronic Measurements & Instrumentation

Question & Answers

8. Explain the terms (i). significant figures (ii). Conformity


Ans: (i) Significant Figures The number of meaningful digits used to express a numerical value (measured value of a quantity) are known as, significant figures. Significant figures indicate the precision of the measurement and the magnitude of the measurements. The measured value should be expressed in more number of significant figures because the more significant figures the higher will be the precision. Consider an example in which the measured voltage across a resistor in a circuit is specified as 50 V. It indicates that the measured voltage may be close to 49 V or 51 V. This specification has two significant figures. If the measured voltage is specified as 50.0 V then it indicates that the value may be close to 49.9 V or 50. 1 V. This specification has three significant figures. From the above illustration, it can be observed that the specification with three significant figures is more precise than the one with two significant figures. (ii) Conformity Conformity is one of the characteristics which determine the precision. If a measuring instrument consistently and repeatedly provides a value as close to the true value (of the measured quantity) as an observer can estimate the true value from its scale reading then this characteristic refers to the conformity of the measurement. Let us consider an example of measuring resistance of a resistor which has a true resistance of 10,654,739 ). If the multi meter indicates the resistance value as 10.7 MO consistently and repeatedly, then the condition of conformity is satisfied. But, due to the limitation of GRIET/ECE

13

www.jntuworld.com

www.jntuworld.com

www.jwjobs.net

Electronic Measurements & Instrumentation

Question & Answers

scale reading, there exists an error in the measured value as the scale provides the reading up to one decimal place only. Thus, conformity is a necessary condition, but not a sufficient condition for the measurement to be precise.

9. What is ayrton shunt? Describe it with a neat sketch .specify its application? Ans: Aryton shunt: It is also known as universal shunt. Figure shows the basic circuit of an aryton shunt.

It avoids the possibility of using the meter in the circuit without a shunt. This is the most important merit of the aryton shunt. From the above figure, it is noted that the series combination of resistors R2, R3 and the meter movement is in parallel with R1 when the switch (SW) is connected to position "1". Therefore, the current through the meter movement is less than the current through the shunt, thereby protecting the meter movement. This reduces the sensitivity of meter movement. The series combination of resistor R6 and the meter movement is in parallel with resistor R1, R2, when the SW is connected to position "2". Therefore, the current through the shunt resistance is less than the current through the meter movement. The resistors R1, R2, and R3, are together in parallel with meter movement. When the switch is in position "3".Now the current flowing through the shunt is very little whereas the current flowing through the meter is very high. Hence the sensitivity of the meter movement is increases. GRIET/ECE 14

www.jntuworld.com

www.jntuworld.com

www.jwjobs.net

Electronic Measurements & Instrumentation

Question & Answers

10. Explain with a neat block diagram of a dual slope digital voltmeter? Ans:
Basic Principle: Initially, the dual slope integrating type DVM integrates the input voltage Vi. The slope of the integrated signal is proportional to the input voltage under measurements .after certain period of time say t1 the supply of input voltage Vi is stopped, and a negative voltage -Vr of the integrator. Then the output signal of integrator will have negative slope, and is constant and also proportional to the magnitude of the input voltage.
BLOCK DIAGRAM AND WORKING:

The major blocks of a dual slope integrating type DVM (dual slope analog to digital converter) are, 1. An op-amp employed as an integrator 2. A level comparator 3. Oscillator for generating time pulses 4. Decimal counter 5. Block of logic circuitry. GRIET/ECE

15

www.jntuworld.com

www.jntuworld.com

www.jwjobs.net

Electronic Measurements & Instrumentation

Question & Answers

Initially a pulse is applied to reset the counter and the output of flip-flop will be at logic '0'. The switch Sr is in open condition and the switch, Si is in closed condition. Now, the capacitor 'C' starts to charge. Once the output of the integrator becomes greater than zero, the output state of the comparator changes which in turn opens the AND gate .When the gate opens the output of the oscillator (clock pulses) are allowed to pass through it and applied to the counter. Now the counter counts the number of pulses fed to it. As soon as it reaches its maximum count that is the counter is preset to run for a time period r,, in this condition the maximum count will be'9999', and for the next immediate clock pulse the count changes or goes to '0000' and the flip-flop will be activated. Therefore, the output of flip flop becomes logic 'I' which in turn activates the switch drive circuitry. This makes the switch Si, to open and Sr to close (i.e., the supply of Vi will be stopped. and the supply of V is applied to the integrator) with this applied signal the output of the integrator will be a constant negative slope i.e., its output signal linearly decreases to zero. This again makes the output of the comparator to change its state which in turn closes the gate. Here, the discharging time t2 of the capacitor is proportional to the input voltage signal Vi .During this discharging period the counter indicates the count. As soon as, the negative slope reaches zero volts the comparator changes its output state to 'zero' which in turn locks the gate. Once, the output of integrator becomes zero (or the input of the comparator is zero) the counter will be stopped. And the counted pulses are displayed (which directly gives the input voltage).

From the above equation, it is clear that the measured voltage signal's accuracy does not depend on the time constant of the integrator. Advantages 1. Depending on the requirement the accuracy and sped can be varied. 2. It can provide the output with an accuracy of +-0.005% in 100ms 3. This technique exhibits excel lent noise rejection since the integration process eliminates both noise and super imposed A.C.

11. Explain the constructional details and differentiate between Ohmmeter type and shunt type. ? Ans: ohmmeter (SERIES TYPE OHM METER)

series

A D'Arsonval movement is connected in series with a resistance R, and a battery which is connected to a pair of terminals A and B, across which the unknown resistance is connected. This forms the basic type of series ohm meter, as shown in the fig 11. GRIET/ECE

16

www.jntuworld.com

www.jntuworld.com

www.jwjobs.net

Electronic Measurements & Instrumentation

Question & Answers

The current flowing through the movement then depends on the magnitude of the unknown resistance. Therefore, the meter deflection is directly proportional to the value of the unknown resistance referring to the figure 11. R1: current limiting resistance R2: zero adjust resistance V= battery Rm =meter resistance Rx=un know resistance Calibration of the Series Type Ohmmeter: To mark the "0" reading on the scale, the terminals A and B are Shorted, i.e. the Unknown resistance Rx=0, maximum current flows in the circuit and the shunt Resistance R2 is adjusted until the movement indicates full scale current (Ifsd ). The Position of the pointer on the scale is then marked "0" ohms. Similarly, to mark the "" reading on the Scale, terminals A and B are open, i.e., the unknown resistance Rx=, no current flow in the circuit and there is no deflection of the pointer. The position of the pointer on the scale is then marked as 0hms. By connecting different known values of the unknown resistance to terminals A and B, intermediate markings can be done on the scale. The accuracy of the Instrument can be checked by measuring different values of standard resistance, i.e., the tolerance of the calibrated resistance, and noting the readings a major drawback in the series ohmmeter is the decrease in voltage of the internal battery with time and age. Due to this, the full scale deflection current Drops and the meter does not read "0" when A and B are shorted. The variable Shunt resistor R2 across the movement is adjusted to counteract the drop in battery Voltage. There by bringing the pointer back to "0" ohms on the scale' It is also possible to adjust the full scale deflection current without the shunt R2 in the circuit, by varying the value of R1, to compensate for the voltage drop. Since this value affects the calibration of the scale, varying by R2 is much better solution. The internal resistance of the coil Rm is very low compared to R1 When R2 is varied, the current through the movement is increased and the GRIET/ECE 17

www.jntuworld.com

www.jntuworld.com

www.jwjobs.net

Electronic Measurements & Instrumentation

Question & Answers

current through R2 is reduced, thereby bringing the pointer to the full scale deflection position. The series ohmmeter is a simple and popular design, and is used extensively For general services work, Therefore ,in a series ohmmeter the scale marking on the dial has 0 on the right side ,corresponding to full scale deflection current ,and "" on the left side corresponding to no current flow as given in the fig 11.1 Values of R1 and R2 can be determined from the value of Rx ,which gives half the full scale deflection.

GRIET/ECE

18

www.jntuworld.com

www.jntuworld.com

www.jwjobs.net

Electronic Measurements & Instrumentation

Question & Answers

UNIT 2

Q.1) Describe the functioning of standard signal generator Ans.


STANDARD SIGNAL GENERATOR A standard signal generator produces known and controllable voltages. It is used as power source for the measurement of gain, signal to noise ratio (SN), bandwidth standing wave ratio and other properties. It is extensively used in the measuring of radio receivers and transmitter instrument is provided with a means of modulating the carrier frequency, which is indicated by the dial setting on the front panel. The modulation is indicated by a meter. The output signal can be Amplitude Modulated (AM) or Frequency Modulated (FM). Modulation may be done by a sine wave, Square, rectangular, or a pulse wave. The elements of a conventional signal generator

GRIET/ECE

www.jntuworld.com

www.jntuworld.com

www.jwjobs.net

Electronic Measurements & Instrumentation

Question & Answers

The carrier frequency is generated by a very stable RF oscillator using an LC tank circuit, having a constant output over any frequency range. The frequency of oscillations is indicated by the frequency range control and the venire dial setting. AM is provided by an internal sine wave generator or from an external source.

Q.2) how can a sine and square wave be generated using signal generator? Ans.
The signal generator is called an oscillator. A Wien bridge oscillator is used in this generator. The Wien bridge oscillator is the best of the audio frequency range. The frequency of oscillations can be changed by varying the capacitance in the oscillator. The frequency can also be changed in steps by switching the resistors of different values. The output of the Wien bridge oscillator goes to the function switch. The function switch directs the oscillator output either to the sine wave amplifier or to the square wave shaper. At the output, we get either a square or sine wave. The output is varied by means of an attenuator.

The instrument generates a frequency ranging from 10 Hz to 1 MHz continuously variable in 5 decades with overlapping ranges. The output sine wave amplitude can be varied from 5 mV to 5 GRIET/ECE 2

www.jntuworld.com

www.jntuworld.com

www.jwjobs.net

Electronic Measurements & Instrumentation

Question & Answers

V (rms).The output is taker through a push-pull amplifier. For low output, the impedance is 6000. The square wave amplitudes can be varied from 0 - 20 v (peak). It is possible to adjust the symmetry of the square wave from 30 -70%. The instrument requires only 7W of power at 220V 50Hz. The front panel of a signal generator consists of the following.

It selects the frequency in different ranges and varies it continuously in a ratio of 1: 11. The scale is non-linear. 2. Frequency multiplier: It selects the frequency range over 5 decades from 10 Hz to 7 MHz 3. Amplitude multiplier: It attenuates the sine wave in 3 decades, x l x 0.1 and x 0.01. 4. Variable amplitude: It attenuates the sine wave amplitude continuously 5. Symmetry control: It varies the symmetry of the square wave from 30% to 70%. 6. Amplitude: It attenuates the square wave output continuously. 7. Function switch: It selects either sine wave or square output. 8. Output available: This provides sine wave or square wave output. 9. Sync: This terminal is used to provide synchronization of the internal signal with an external signal. 10. On-Off Switch

l. Frequency selector:

Q.3) Explain how a Function Generator works? Ans:


FUNCTION GENERATOR A function generator produces different waveforms of adjustable frequency. The common output waveforms are the sine, square, triangular and saw tooth waves. The frequency may be adjusted, from a fraction of a Hertz to several hundred kHz lie various outputs of the generator can be made available at the same time. For example, the generator can provide a square wave to test the linearity of a rectifier and simultaneously provide a saw tooth to drive the horizontal deflection amplifier of the CRO to provide a visual display. Capability of Phase Lock the GRIET/ECE 3

www.jntuworld.com

www.jntuworld.com

www.jwjobs.net

Electronic Measurements & Instrumentation

Question & Answers

function generator can be phase locked to an external source. One function generator can be used to lock a second function generator, and the two output signals can be displaced in phase by adjustable amount. In addition, the fundamental frequency of one generator can be phase locked to a harmonic of another generator, by adjusting the amplitude and phase of the harmonic; almost any waveform can be generated by addition. The function generator can also be phase locked to a frequency standard and its output waveforms will then have the same accuracy and stability as the standard source. The block diagram of a function generator is illustrated in fig. Usually the frequency is controlled by varying the capacitor in the LC or RC circuit. In the instrument the frequency is controlled by varying the magnitude of current which drives the integrator. The instrument produces sine, triangular and square waves with a frequency range of 0.01 Hz to 100 kHz.

The frequency controlled voltage regulates two current sources. The upper current source supplies constant current to the integrator whose output voltage increases linearly with time, according to the equation of the output signal voltage. An increase or decrease in the current increases or decreases the slope of the output voltage and hence controls the frequency. The GRIET/ECE 4

www.jntuworld.com

www.jntuworld.com

www.jwjobs.net

Electronic Measurements & Instrumentation

Question & Answers

voltage comparator multi-vibrator changes states at a pre-determined maximum level of the integrator output voltage. This change cuts off the upper current supply and switches on the lower current supply. The lower current source supplies a reverse current to the integrator, so that its output decreases linearly with time. When the output reaches a pre-determined minimum level, the voltage comparator again changes state and switches on the Lower current source. The output of the integrator is a triangular waveform whose frequency is determined by the magnitude of the current supplied by the constant current sources. The comparator output delivers a square wave voltage of the same frequency.

e=-

The resistance diode network alters the slope of the triangular wave as its amplitude changes and produces a sine wave with less than 1% distortion.

Q.4) Explain the functioning of Random Noise Generator and explain the parameters of noise? Ans:

RANDOM NOISE GENERATOR

GRIET/ECE

www.jntuworld.com

www.jntuworld.com

www.jwjobs.net

Electronic Measurements & Instrumentation

Question & Answers

The spectrum of random noise covers all frequencies the lower density spectrum tells us how the energy of the signal is distributed in frequency, but it does not specify the signal uniquely nor does it tell us very much about how the amplitude of the signal varies with time The spectrum does not specify the signal uniquely because it contains no phase information. The method of generating noise is usually to use a semi conductor noise which delivers frequencies in a band roughly extending from 80 220 KHz The output from the noise diode is amplified and heterodyned down to audio frequency band by means of a balanced symmetrical modulator. The filter arrangement controls the bandwidth and supplies an output signal in three spectrum choices, white noise, pink noise and Usasi noise. From the Fig 4.2 it is seen that white noise is flat from 20Hz to 20 KHz and has upper cutoff frequency of 50 kHz with a cutoff slope of -12 dbs/ octave. Pink noise is so called because the lower frequencies have larger amplitude, similar to red light. Pink noise has a voltage spectrum which is inversely proportional to the square root of frequency and is used in band analysis. Usasi noise ranging simulates the energy distribution of speech and music frequencies and is used for testing audio amplifiers and loud speakers.

GRIET/ECE

www.jntuworld.com

www.jntuworld.com

www.jwjobs.net

Electronic Measurements & Instrumentation

Question & Answers

Q.5 what is a Sweep Generator, explain its functioning? Ans:


It provides a sinusoidal output voltage whose frequency varies smoothly and continuously over an entire frequency band, usually at an audio rate. The process of frequency modulation may be accomplished electronically or mechanically. It is done electronically by using the modulating voltage to vary the reactance of the oscillator tank circuit component, and mechanically by means of a motor driven capacitor, as provided for in a modern laboratory type signal generator. Figure shows a basic block diagram of a sweep generator. The frequency sweeper provides a variable modulating voltage which causes the capacitance of the master oscillator to vary. A representative sweep rate could be of the order of 20 sweeps/second. A manual control allows independent adjustment of the oscillator resonant frequency. The frequency sweeper provides a GRIET/ECE 7

www.jntuworld.com

www.jntuworld.com

www.jwjobs.net

Electronic Measurements & Instrumentation

Question & Answers

varying voltage

sweep for

synchronization to drive the horizontal deflection plates of the CRO. Thus the amplitude of the response of a test device will be locked and displayed on the screen.

GRIET/ECE

www.jntuworld.com

www.jntuworld.com

www.jwjobs.net

Electronic Measurements & Instrumentation

Question & Answers

To identify a frequency interval, a marker generator provides half sinusoidal waveforms at any frequency within the sweep range. The marker voltage can be added to the sweep voltage of the CRO during alternate cycles of the sweep voltage, and appears superimposed on the response curve. The automatic level control circuit is a closed loop feedback system which monitors the RF level at some point in the measurement system. This circuit holds the power delivered to the load or test circuit constant and independent o frequency and impedance changes. A constant power level prevents any source mismatch and also provides a constant readout calibration with frequency.

6. Explain Square and Pulse Generator?


ANS: SQUARE AND PULSE GENERATOR: These generators are used as measuring devices in combination with a CRO. They provide both quantitative and qualitative information of the system under test. They are made use of in transient response testing of amplifiers. The fundamental difference between a pulse generator and a square wave generator is in the duty cycle.

Duty cycle = A square wave generator has a 500/o duty cycle. Requirements of a Pulse 1. The pulse should have minimum distortion, so that any distortion, in the display is solely due to the circuit under test. 2. The basic characteristics of the pulse are rise time, overshoot, ringing, sag, and undershoot. 3. The pulse should have sufficient maximum amplitude, if appreciable output power is required by the test circuit, e.g. for magnetic core memory. At the same time, the attenuation range should be adequate to produce small amplitude pulses to prevent over driving of some test circuit. 4. The range of frequency control of the pulse repetition rate (PRR) should meet the needs of the experiment. For example, a repetition frequency of 100 MHz is required for testing fast circuits. GRIET/ECE 9

www.jntuworld.com

www.jntuworld.com

www.jwjobs.net

Electronic Measurements & Instrumentation

Question & Answers

Other generators have a pulse-burst feature which allows a train of pulses rather than a continuous output. 5. Some pulse generators can be triggered by an externally applied trigger signal; conversely, pulse generators can be used to produce trigger signals, when this output is passed through a differentiator circuit. 6. The output impedance of the pulse generator is another important consideration. In a fast pulse system, the generator should be matched to the cable and the cable to the test circuit. A mismatch would cause energy to be reflected back to the generator by the test circuit, and this may be rereflected by the generator, causing distortion of the pulses. 7. DC coupling of the output circuit is needed, when dc bias level is to be maintained. The basic circuit for pulse generation is the asymmetrical multi-vibrator. A laboratory type square wave and pulse generator is shown in Fig 6.1 The frequency range of the instrument is covered in seven decade steps from 1Hz to 10 MHz, with a linearly calibrated dial for continuous adjustment on all ranges.

The duty cycle can be varied from 25 - 75%. Two independent outputs are available, a 50 source that supplies pulses with a rise and fall time of 5 ns at 5V peak amplitude and a 600 source which supplies pulses with a rise and fall tme of 70 ns at 30 V peak amplitude. The GRIET/ECE 10

www.jntuworld.com

www.jntuworld.com

www.jwjobs.net

Electronic Measurements & Instrumentation

Question & Answers

instrument can be operated as a freerunning genenrator or, it can be synchronized with external signals. The basic generating loop consists of the current sources, the ramp capacitor, the Schmitt trigger and the current switching circuit as shown in the fig 6.2

The upper current source supplies a constant current to the capacitor and the capacitor voltage increases linearly. When the positive slope of the ramp voltage reaches the upper limit set by the internal circuit components, the Schmitt trigger changes state. The trigger circuit output becomes negative and reverses the condition of the current switch. The capacitor discharges linearly, controlled by the lower current source. When the negative ramp reaches a predetermined lower level, the Schmitt trigger switches back to its original state. The entire process is then repeated. The ratio i1/i2 determines the duty cycle, and is controlled by symmetry control. The sum of i1 and i2 determines the frequency. The size of the capacitor is selected by the multiplier switch. The unit is powered by an intenal supply that provides regulated voltages for all stages of the instrument.

7. What is the basic difference between a signal generator and an oscillator? Discuss fixed and variable AF oscillator? Ans:
Signal generators are the sources of electrical signals used for the purpose of testing and operating different kinds of electrical equipment. A signal generator provides different types of waveforms such as sine, triangular, square, pulse etc., whereas an oscillator provides only sinusoidal signal at the output. The AF oscillators are divided into two types. They are as follows, GRIET/ECE

11

www.jntuworld.com

www.jntuworld.com

www.jwjobs.net

Electronic Measurements & Instrumentation

Question & Answers

1. Fixed frequency AF oscillator 2. Variable frequency AF oscillator. 1. Fixed Frequency AF Oscillator Many instrument circuits contain oscillator as one of its integral parts to provide output signal within the specified fixed audio frequency range. This specified audio frequency range can be 1 kHz signal or 400 Hz signal. The 1 kHz frequency signal is used to execute a bridge circuit and 400 Hz frequency signal is used for audio testing. A fixed frequency AF oscillator employs an iron core transformer. Due to this a positive feedback is obtained through the inductive coupling placed between the primary winding and secondary winding of the transformer and hence fixed frequency oscillations are generated. 2. Variable Frequency AF Oscillator It is a general purpose oscillator used in laboratory. It generates oscillations within the entire audio frequency range i.e. from 2O Hz to 20 kHz. This oscillator provides a pure, constant sine wave output throughout this AF range. The examples of variable AF oscillators used in laboratory are RC feedback oscillator, beat frequency oscillator. 8. With a neat block diagram discuss about an AF sine wave generator? Ans: As the name suggest an AF sine and square wave generator produces either sine wave or square wave output. It employee a Wein bridge oscillator, sine wave amplifier, square wave shaper, square wave amplifier and attenuator. The schematic arrangement of these blocks is shown below. The Wien bridge oscillator operates effectively in audio frequency ranges. It produces oscillations whose frequency can be varied by varying the capacitance value of the capacitor of the oscillator. Also the frequency value can be varied in steps by switching in different values of resistors. The oscillations of Wien bridge oscillator are applied to either sine wave amplifier or sine wave shaper through function key. When the key is connected to position 1, the output oscillations are connected to sine wave amplifier and then to attenuator. Therefore, the oscillations are amplified and then attenuated and a pure sine wave is available at the output. Depending on the requirement the amplitude of this sine wave can be varied from 5 mV to 5 V (r.m.s value). When the key is connected to position 2, the oscillations are applied to square wave shaper which converts the oscillations into square wave. The square wave signal is amplified and then attenuated and finally appears as pure square wave at the output. The amplitude of the square wave can be varied from 0 V to 20 V (peak value). GRIET/ECE 12

www.jntuworld.com

www.jntuworld.com

www.jwjobs.net

Electronic Measurements & Instrumentation

Question & Answers

This generator produces output in the frequency range of 1 0 Hz to I MHz and it requires power of 7 W at 220V, 50 Hz. The front panel of the instrument contains (i) ON/OFF switch. (ii)FrequencyMultip1ier: To choose the frequency range over 5 decades (from10Hz to1MHz). (iii) Amplitude Multiplier: To attenuate sine wave output in 3 decades (x 1; x 0.1 and x 0.01). (iv) Amplitude: To continuously attenuate the amplitude of square wave output. (v) Variable Amplitude: To continuously attenuate the amplitude of sine wave output. (vi) Frequency Selector: To select different ranges of frequencies and to vary the frequency in a ratio of 1: 11. (vii) Function Key: To select either square wave or sine wave output. (viii) Symmetry Control: To adjust the symmetry of square wave from 30% to 70% (ix) Sync: To synchronize the internal signal with external signal.

8. What is need for inserting isolation between the signal generator output and oscillator in a simple signal generator? What are the difference ways in which this can be achieved? Ans: An oscillator of a simple signal generator needs to be isolated from the output of the signal generator because any variations in the load (output circuit of signal generator) will affect the output characteristic (i.e., amplitude, frequency, etc) of an oscillator. Usually, the frequency of an oscillator should be very stable when the oscillator is operating at high frequencies of the order of MHz, because even a small variation in the frequency will give rise to errors. Hence, an isolation of 20 dB or more (based upon the type of oscillator circuit) should be introduced between oscillator and signal generator output. The different ways to achieve an isolation of 20 dB or more between oscillator and signal generator output are, 1. Setting the attenuation of attenuator to 20 dB or more. 2. Introducing an isolation amplifier between oscillator and attenuator 1. Setting the Attenuation of Attenuator to 20 dB or More In a signal generator the output of the oscillator is attenuate d by feeding it to variable attenuator, in order to obtain a signal of desired amplitude (or power level). So, if the attenuator is set to provide and attenuate the oscillator output by 20 dB or more, an isolation of 20 dB will be produced between the oscillator and the load. GRIET/ECE 13

www.jntuworld.com

www.jntuworld.com

www.jwjobs.net

Electronic Measurements & Instrumentation

Question & Answers

2. Introducing an Isolation Amplifier between oscillator and Attenuator In this method of achieving isolation, the oscillator output is amplified by certain amount using a buffer amplifier. Consecutively, it is attenuated by same amount by a fixed attenuator before feeding the oscillator output to the variable attenuator of the signal generator. In this way isolation is achieved without any change in the signal level of oscillator output. To achieve an isolation of 20 dB or more, a 10 dB gain isolation amplifier followed by a 10 dB fixed attenuator is introduced between the oscillator and variable attenuator as shown in the following figure. The gain of the isolation amplifier and thus the attenuation of a fixed attenuator depend on the amount of isolation required and also on the attenuation of the variable attenuator

9. with respect to construction and circuit configuration, explain how a square wave generator differs from sine wave generator? Ans:
Sine Wave Generator The circuit configuration of a sine wave generator consists of Wien bridge oscillator, sine wave amplifier and attenuator. The block diagram of a sine wave generator is shown in figure 9.1

GRIET/ECE

14

www.jntuworld.com

www.jntuworld.com

www.jwjobs.net

Electronic Measurements & Instrumentation

Question & Answers

Wein bridge oscillator produces an oscillating output which is usually a sinusoidal (sine) wave. Thus, half of the operation of a sine wave generator is done by the Wein bridge oscillator. The frequency of oscillations of this oscillator can be varied by varying its capacitance and thus a sine wave of desired frequency can be generated. The remaining elements of sine wave generator i.e., amplifier, and attenuator are used as signal conditioners to condition the output of Wien bridge oscillator in order to obtain a sine wave of desired amplitude. Square Wave Generator The circuit configuration of a square wave generator consists of the basic elements of a sine wave generator (i.e., Wien bridge oscillator, attenuator) and square wave shaper and square wave amplifier. Figure 9.2 shows the block diagram of a square wave generator.

A square wave is obtained by feeding the sinusoidal output of the Wein bridge oscillator to the square wave Shaper circuit. The square wave shaper is usually a sine-to-square wave converter. The square wave is further processed through square wave amplifier and attenuator in order to obtain a square wave of desired amplitude. The frequency of the square wave can be varied by varying the oscillation frequency of Wein bridge oscillator.

10. What are the precautionary measures to bee taken in a signal generator application?
GRIET/ECE 15

www.jntuworld.com

www.jntuworld.com

www.jwjobs.net

Electronic Measurements & Instrumentation

Question & Answers

Ans; A signal generator is an instrument, which can produce various types of wave forms such as sine wave, square wave, triangular wave, saw tooth wave, pulse trains etc. As it can generate a variety of waveforms it is widely used in applications like electronic troubleshooting anti development, testing the performance of electronic equipments etc. In such applications a signal generator is used to provide known test conditions (i.e., desired signals of known amplitude and frequency Hence, the following precautionary measures should be taken while using a signal generator for an application. 1. The amplitude and frequency of the output of the signal generator should be made stable and well known. 2. There should be provision for controlling the amplitude of signal generator output from very small to relatively large values. 3. The output signal of generator should not contain any distortion and thus, it should possess very low harmonic contents. 4. Also, the output of the signal generator should be less spurious.

GRIET/ECE

16

www.jntuworld.com

www.jntuworld.com

www.jwjobs.net

Electronic Measurements & Instrumentation

Question & Answers

UNIT-3

1. Draw the Block Schematic of AF Wave analyzer and explain its principle and Working? ANS: The wave analyzer consists of a very narrow pass-band filter section which can
Be tuned to a particular frequency within the audible frequency range(20Hz to 20 KHz)). The block diagram of a wave analyzer is as shown in fig 1.

The complex wave to be analyzed is passed through an adjustable attenuator which serves as a range multiplier and permits a large range of signal amplitudes to be analyzed without loading the amplifier. GRIET/ECE1

www.jntuworld.com

www.jntuworld.com

www.jwjobs.net

Electronic Measurements & Instrumentation

Question & Answers

The output of the attenuator is then fed to a selective amplifier, which amplifies the selected frequency. The driver amplifier applies the attenuated input signal to a high-Q active filter. This high-Q filter is a low pass filter which allows the frequency which is selected to pass and reject all others. The magnitude of this selected frequency is indicated by the meter and the filter section identifies the frequency of the component. The filter circuit consists of a cascaded RC resonant circuit and amplifiers. For selecting the frequency range, the capacitors generally used are of the closed tolerance polystyrene type and the resistances used are precision potentiometers. The capacitors are used for range changing and the potentiometer is used to change the frequency within the selected pass-band, Hence this wave analyzer is also called a Frequency selective voltmeter. The entire AF range is covered in decade steps by switching capacitors in the RC section . The selected signal output from the final amplifier stage is applied to the meter circuit and to an unturned buffer amplifier. The main function of the buffer amplifier is to drive output devices, such as recorders or electronics counters. The meter has several voltage ranges as well as decibel scales marked on it. It is driven by an average reading rectifier type detector. The wave analyzer must have extremely low input distortion, undetectable by the analyzer itself. The band width of the instrument is very narrow typically about 1% of the selective band given by the following response characteristics shows in fig.1.2

2. What are the applications of wave Analyzer? Ans: Application of wave analyzer
1. Electrical measurements 2. Sound measurements 3. Vibration measurements. In industries there are heavy machineries which produce a lot of sound and vibrations, it is very important to determine the amount of sound and vibrations because if it exceeds the permissible GRIET/ECE2

www.jntuworld.com

www.jntuworld.com

www.jwjobs.net

Electronic Measurements & Instrumentation

Question & Answers

level it would create a number of problems. The source of noise and vibrations is first identified by wave analyzer and then it is reduced by further circuitry.

3. Explain the working of the harmonic distortion analyzer? Ans:


Fundamental Suppression Type Distortion analyzer measures the total harmonic power present in the test wave rather than the distortion caused by each component. The simplest method is to suppress the fundamental frequency by means of a high pass filter whose cut off frequency is a little above the fundamental frequency. This high pass allows only the harmonics to pass and the total harmonic distortion can then be measured. Other types of harmonic distortion analyzers based on fundamental suppression are as follows 1. Employing a Resonance Bridge T h e b r i d g e s h o w n in fig 3.1 is balanced for the fundamental frequency, i.e. L and C are tuned to the fundamental frequency. The bridge is unbalanced for the harmonics, i.e. only harmonic power will be available at the output terminal and can be measured. If the fundamental frequency is changed, the bridge must be balanced again. If L and CCCCCCCCCCCCCCCCCC are fixed components, then this method is suitable only when the test wave has a fixed frequency. Indicators can be thermocouples or square law VTVMs. This indicates the rms value of all harmonics. When a continuous adjustment of the fundamental frequency is desifrequency is desired a Wien bridge arrangement is used as shown in fig 3.2.

GRIET/ECE3

www.jntuworld.com

www.jntuworld.com

www.jwjobs.net

Electronic Measurements & Instrumentation

Question & Answers

2. Wiens Bridge Method The bridge is balanced for the fundamental frequency. The fundamental energy is dissipated in the bridge circuit elements. Only the harmonic components reach the output terminals .The harmonic distortion output can then be measured with a meter. For balance at the fundamental frequency C1=C2=C, R1=R2=R, R3=2R4.

3. Bridged T-Network Method Referring to the fig 3.3 the L and Cs are tuned to the fundamental frequency, and R is adjusted to bypass fundamental frequency. The tank circuit being tuned to the fundamental frequency, the fundamental energy will circulate in the tank and is bypassed by the resistance. Only harmonic components will reach the output terminals and the distorted output can be measured by the meter. The Q of the resonant circuit must be at least 3-5.

One way of using is given in Fig. 3.4 connected to point attenuator is bridge T-network is suppression of the frequency, i.e. indicates that the network is tuned to fundamental and that fundamental is fully suppressed.

a bridge T-network The switch S is first A so that the excluded and the adjusted for full fundamental Minimum output bridged Tthe frequency frequencies

GRIET/ECE4

www.jntuworld.com

www.jntuworld.com

www.jwjobs.net

Electronic Measurements & Instrumentation

Question & Answers

The switch is next connected to terminal B, i.e. the bridge T- network is excluded. Attenuation is adjusted until the same reading is obtained on the meter. The attenuator reading indicates the total rams distortion. Distortion measurement can also be obtained by means of a wave analyzer, knowing the amplitude and the frequency of each component, the harmonic distortion can be calculated. However, distortion meters based on fundamental suppression are simpler to design and less expensive than wave analyzers. The is advantage is that 1give only the total distortion and not the amplitude of individual distortion components.

4. Draw the block Schematic of a Basic Spectrum Analyzer and explain its working? Ans: The most common way of observing signals is
to display them on an oscilloscope with time as the X-axis (i.e. amplitude of the signal versus time). This is the time domain. It is also useful to display signals in the frequency domain. The providing this frequency domain view is the spectrum analyzer. A spectrum analyzer provides a calibrated graphical display on its CRT, with frequency on the horizontal axis and amplitude (voltage) on the vertical axis.

GRIET/ECE5

www.jntuworld.com

www.jntuworld.com

www.jwjobs.net

Electronic Measurements & Instrumentation

Question & Answers

Displayed as vertical lines against these coordinates are sinusoidal components of which the input signal is composed. The height represents the absolute magnitude, and the horizontal location represents the frequency. These instruments provide a display of the frequency spectrum a given frequency band. Spectrum analyzers use either parallel filter bank or a swept frequency technique. In a parallel filter in a parallel filter bank analyzer, The frequency range is covered by a series of filters whose central frequencies and bandwidth are so selected that they overlap each others, as shown in fig 4.1. Typically, an audio analyzer has 32 of these filters, each covering one third of an octave. For wide band narrow resolution analysis, particularly at RF or microwave signals, the swept Technique is preferred.

Basic Spectrum Analyzer Using Swept Receiver Design Referring to the block diagram of fig. 4.2, the saw tooth generator provides the saw tooth voltage which drives the horizontal axis element of the scope and this saw tooth voltage is the frequency controlled element of the voltage tuned oscillator. As the oscillator sweeps from fmin to fmax of its frequency band at a linear recurring rate, it beats with the frequency component of the input signal and produce an IF, whenever a frequency component is met during its sweep. The frequency component and voltage tuned oscillator frequency beats together to produce a difference frequency, i.e. The IF corresponding to the component is amplified and detected if necessary and then applied to the vertical plates of the CRO, producing a display of amplitude versus frequency.

GRIET/ECE6

www.jntuworld.com

www.jntuworld.com

www.jwjobs.net

Electronic Measurements & Instrumentation

Question & Answers

The spectrum produced if the input wave is a single toned A.M is given in figs 4.3, 4.4 and 4.5

GRIET/ECE7

www.jntuworld.com

www.jntuworld.com

www.jwjobs.net

Electronic Measurements & Instrumentation

Question & Answers

One of the principal applications of spectrum analyzers has been in the study of the RF spectrum produced in microwave instruments. In a microwave instrument, the horizontal axis can display as a wide a range as 2 - 3 GHz for a broad survey and as narrow as 30 kHz, for a highly magnified view of any small portion of the spectrum. Signals at microwave frequency separated by only a few KHz can be seen individually. The frequency range covered by this instrument is from I MHz to 40 GHz, The basic block diagram (Fig. 9.13) is of a spectrum analyzer covering the range 500 kHz to 1 GHz, which is representative of a super heterodyne type.

GRIET/ECE8

www.jntuworld.com

www.jntuworld.com

www.jwjobs.net

Electronic Measurements & Instrumentation

Question & Answers

The input signal is fed into a mixer which is driven by a local oscillator. This oscillator is linearly tunable electrically over the range 2 - 3 GHz. The mixer provides two signals at its output that are proportional in amplitude to the input signal but of frequencies which are the sum and difference of the input signal and local oscillator frequency. The IF amplifier is tuned to a narrow band around 2 GH4 since the local oscillator is tuned over the range of 2 - 3 GHz, only inputs that are separated from the local oscillator frequency by 2GHz will be converted to IF frequency band, pass through the IF frequency amplifier, get rectified and produce a vertical deflection on the CRT. From this, it is observed that as the saw tooth signal sweeps, the local oscillator also sweeps linearly from 2 - 3 GHz. The tuning of the spectrum analyzer is a swept receiver, which sweeps linearly from 0 to 1 GHz. The saw tooth scanning signal is also applied to the horizontal plates of the CRT to form the frequency axis. (The spectrum analyzer is also sensitive to signals from 4 5 GHz referred to as the image frequency of the super heterodyne. A low pass filter with a cutoff frequency above I GHz at the input suppresses these spurious signals.) Spectrum analyzers are widely used in radars, oceanography, and bio-medical fields

5. With a neat sketch explain the working of a digital Fourier analyzer? An: A spectrum analyzer, which uses computer algorithm and an analog to digital conversion
phenomenon and produces spectrum of a signal applied at its input is known as digital Fourier or digital FFT or digital spectrum analyzer. GRIET/ECE9

www.jntuworld.com

www.jntuworld.com

www.jwjobs.net

Electronic Measurements & Instrumentation

Question & Answers

Principle When the analog signal to be analyzed is applied, the A/D converter digitizes the analog signal (i.e., converts the analog signal into digital signal). The digitized signal, which is nothing but the set of digital numbers indicating the amplitude of the analog signal as a function of time is stored in the memory of the digital computer. From the stored digitized data, the spectrum of the signal is computed by means of computer algorithm. Description: The block arrangement of a digital Fourier analyzer is illustrated in the figure above fig 5.The analog signal to be ana1ysed is applied to the low pass filter, which passes only low frequency signals and rejects high pass spurious signals. This filter section is used mainly, to prevent aliasing. The output of low pass filter is given to the attenuator. The attenuator is a voltage dividing network whose function is to set the input signal to the level of the A/D converter. The use of attenuator prevents the converter from overloading. The function of A/D converter is to convert the samples of analog data into digital i.e. ., to digitize the analog signal. When the output of A/D converter is applied to the digital computer, the computer analyzes the digitized data and adjusts the attenuator setting accordingly in order to obtain the maximum output from the inverter without any overloading. As soon as the entire analog signal is sampled and digitized by the A/D converter) computer performs calculations on the data according to the programmed algorithm and the calculated spectral components are stored in the memory of the computer.

GRIET/ECE10

www.jntuworld.com

www.jntuworld.com

www.jwjobs.net

Electronic Measurements & Instrumentation

Question & Answers

If the spectral display is to be viewed on the oscilloscope, the digital values of spectral components stored in the computer memory are converted into analog by using D/A converters and then applied to the CRO. Thus the spectral display of the input waveform is obtained on the CRT screen. Advantages 1. The use of computer avoids most of the hardware circuitry such as electronic switches. Filters and PLLs. The use of less hardware reduces the cost of the analyzer. 2. More mathematical calculations can be carried-out on the spectral display. 3. The rate of sampling analog signal can be modified in order to obtain better spectral display.

6. Differniate between wave analyzer and harmonic distortion analyzer?


GRIET/ECE11

www.jntuworld.com

www.jntuworld.com

www.jwjobs.net

Electronic Measurements & Instrumentation

Question & Answers

Ans: Wave analyzer


1. These are designed to measure the relative amplitude of each harmonic or fundamental components separately. 2.They indicate the amplitude of single frequency component

Harmonic distortion analyzer


1. These are designed to measure the total harmonic content present in a distorted or complex wave form. 2. They do not indicate the amplitude of single frequency component

3.These are tuned to measure amplitude of one 3.These can be operated with in a band of 5Hz frequency component with in a range of 10Hz to 1 MHz frequency to 40MHz 4.These are also known as frequency selective voltmeters, selective level voltmeters, carrier frequency voltmeters 5. These are used with a set of tuned filters and a voltmeter. 4.It is general know as distortion analyzer

5. These can be used along with a frequency generator.

6. Wave analyzers provide very high frequency 6. They measure quantitative harmonic resolution. distortions very accurately.7. 7.These can be used for electrical measurements, sound ,vibration ,noise measurement in industries 7.These can be used to measure frequency stability and spectral purity of signal sources

7. Explain the two types of spectrum analyzers? Ans:


The two types of spectrum analyzers are, 1. Fliter Bank Spectrum analyzer. 2. Super hetero dyne Spectrum analyzer. 1. Filter Bank Spectrum analyzer

GRIET/ECE12

www.jntuworld.com

www.jntuworld.com

www.jwjobs.net

Electronic Measurements & Instrumentation

Question & Answers

2. Super hetero dyne Spectrum analyzer The modern spectrum analyzers use a narrow band super heterodyne receiver. Super heterodyne is nothing but mixing of frequencies in the super above audio range. The functional block diagram of super heterodyne spectrum analyzer or RF spectrum analyzer as shown in the Figure 7.2

GRIET/ECE13

www.jntuworld.com

www.jntuworld.com

www.jwjobs.net

Electronic Measurements & Instrumentation

Question & Answers

The RF input to be analyzed is applied to the input attenuator. After attenuating, the signal is fed to low pass filter. The low pass filter suppresses high frequency components and allows low frequency components to pass through it. The output of the low pass filter is given to the mixer, where this signal is fixed with the signal coming from voltage controlled or voltage tuned oscillator. This oscillator is tuned over 2 to 3 GHz range. The output of the mixer includes two signals whose amplitudes. are proportional to the input signal but their frequencies are the sum and difference of the input signal and the frequency of the local oscillator. Since the frequency range of the oscillator is tuned over 2 to 3 GHz, the IF amplifier is tuned to a narrow band of frequencies of about 2 GHz. Therefore only those signals which are separated from the oscillator frequency by 2 GHz are converted to Intermediate Frequency (IF) band. This IF signal is amplified by IF amplifier and then rectified by the detector. After completing amplification and rectification the signal is applied to vertical plates of CRO to produce a vertical deflection on the CRT screen. Thus, when the saw tooth signal sweeps, the oscillator also sweeps linearly from minimum to maximum frequency range i.e., from 2 to 3 GHz. Here the saw tooth signal is applied not only to the oscillator (to tune the oscillator) but also to the horizontal plates of the CRO to get the frequency axis or horizontal deflection on the CRT screen. On the CRT screen the vertical axis is calibrated in amplitude and the horizontal axis is calibrated in frequency. Application: These analyzers are widely used in the field of, 1. Bio medicals GRIET/ECE14

www.jntuworld.com

www.jntuworld.com

www.jwjobs.net

Electronic Measurements & Instrumentation

Question & Answers

2. RADARS 3.Oceangraphy

8. Explain the terms (i) Distortion in a waveform (ii) Distortion in a communication sign
ANS: Distortions may be introduced in a waveform or a communication signal when they are transmitted from one point to other point through a transmission channel. The transmission channel consists of various electronic components like amplifier, heterodyning element etc. The different types of distortions that occur during transmission of an input signal (waveform or communication signal) are, 1. Linear distortions (i) Amplitude distortion (ii) Phase or delay distortion 2. Non-linear distortions (i) Harmonic distortion (ii) Inter modulation distortion.

l. Linear Distortions (i) Amplitude Distortion When different frequency components of the input signal are amplified or attenuated by different amounts, The output signal consists of distortions, known as amplitude distortions (i.e.,) amplitude distortion occurs When the amplification or attenuation of the signal is not constant over the useful range of frequencies. (ii) Phase or Delay Distortion If the phase of the output signal is different from the phase of input signal then such distortion is known as phase distortion. Phase distortion leads to delay in the transmission of the signal. Hence, it is known as delay distortion. If different amounts of phase shifts occur at different frequencies of an output signal then it becomes necessary to compensate for such phase distortions. Whereas if same amount of phase shift occurs at all frequencies then such phase distortion can be ignored. The phase distortion arises due to the presence of energy storage elements in the transmitting circuit (i.e. reactive elements such as capacitor and inductor).

2. Nonlinear Distortions GRIET/ECE15

www.jntuworld.com

www.jntuworld.com

www.jwjobs.net

Electronic Measurements & Instrumentation

Question & Answers

The non-linear distortions in the signal arise due to non-linear characteristics of the electronic components like amplifiers, etc. The two types of non-linear distortions are, (i) Harmonic Distortion

The non-linear characteristics of an electronic circuit give rise to harmonics in the output signal. These harmonics produce distortions in the output signal. The distortions caused due to harmonics are known as harmonic distortions. Harmonic components occur at frequencies 2f1, 3f1, 4f1. (where. f1= Fundamental frequency of signal). (ii) Inter modulation Distortion When two signals of different frequencies (f1 and f2) are mixed together (i.e...heterodyned) the resultant signal will be a sum or difference of the actual frequencies of the signal i.e. f1 f2, 2f1 f2.....etc. Thus, when the signals are heterodyned additional frequency components are generated which are undesirable and which lead to distortions in the signal. The distortion caused by heterodyning of different frequency signals is known as inter modulation distortion.

9. Explain how distortion occurs during transmission of a waveform or Ans


Distortion refers to the deviation in any parameter (like amplitude, frequency. shape) of a signal from that of an ideal signal. The non-linear characteristics of the elements of an electronic circuit give rise to harmonics in the output signal which in turn causes distortion of the output signal. The distortion caused due to harmonics is known as harmonic distortion. The different types of harmonic distortions caused by an electronic circuit (for example, electronic amplifier are as follows, (i) Amplitude distortion (ii) Frequency distortion (iii) Phase distortion (iv) Crossover distortion (v) Inter modulation distortion. (i) Amplitude Distortion When the amplitude of the output signal is not a linear function of the amplitude and input signal is distorted under specific conditions then such type of distortion are known as amplitude distortion. Amplitude distortion occurs when the amplifier gives rise to harmonics of the fundamental frequency of the input signal. (ii) Frequency distortion GRIET/ECE16

www.jntuworld.com

www.jntuworld.com

www.jwjobs.net

Electronic Measurements & Instrumentation

Question & Answers

Frequency distortion of a signal takes place when the signal is amplified by different amounts at different frequencies. This is caused mainly due to the combination of active devices and components in an amplifier. For Example, the non uniform frequency response of RC-coupled cascade amplifier refers to frequency distortion (iii) Phase Distortional: Is different from the phase of the input signal then such distortion is known as phase distortion. If different amounts of phase shifts occur at different frequencies of an output signal than it becomes necessary to compensate for such phase distortions. While if same amount of phase shift occurs at all frequencies then such phase distortion cannot be ignored .the phase distortion arises due to presence of storage elements in the circuit (iv)Crossover Distortion The improper biasing voltages of the electromagnetic components of an amplifier (for example push-pull amplifier give rise to crossover distortion) (v) Inter modulation Distortion When two signals of different frequencies are mixed together (i.e., heterodyned) the resultant signal will be a sum or difference of the actual frequencies of the signals. Thus, when the signals are heterodyned, additional frequencies are generated which are undesirable and thereby leads to distortion. The distortion caused by heterodyning of frequencies is known as inter modulation distortion. The various distortions in the signal can be analyzed using a distortion analyzer (for example, harmonic distortion analyzer).

GRIET/ECE17

www.jntuworld.com

www.jntuworld.com

www.jwjobs.net

Electronic Measurements & Instrumentation


Question & Answers

UNIT-4

1. Explain briefly the Basic Features of a CRT? Ans:


Electrostatic CRTs are available in a number of types and sizes to suit individual requirements. The important features of these tubes are as follows. 1. Size: Size refers to the screen diameter. CRTs for oscilloscopes are available in sizes of 1, 2, 3, 5, and 7 inches. 3 inches is most common for portable instruments For example a CRT having a number 5GPI . The first number 5 indicates that it is a 5 inch tube. Both round and rectangular CRTs are found in scopes today. The vertical viewing size is 8 cm and horizontal is l0 cm. 2. Phosphor: The screen is coated with a fluorescent material called phosphor. This material determines the color and persistence of the trace, both of which are indicated by the phosphor. The trace colors in electrostatic CRTs for oscilloscopes ale blue, green and But green. White is used in TVs. and blue-white, orange, and yellow are used for radar Persistence is expressed as short, medium and long. This refers to the length of time the trace remains on the screen after the signal has ended. The phosphor of the oscilloscope is designated as follows. Pl --Green medium P2--Blue green medium P5--Blue very short P11--Blue short These designations are combined in the tube type number. Hence 5GPl is a 5 inch tube with a medium persistence green trace. Medium persistence traces are mostly used for general purpose applications Long persistence traces are used for transients, since they keep the fast transient on the screen for observation after the transient has disappeared. GRIET/ECE

www.jntuworld.com

www.jntuworld.com

www.jwjobs.net

Electronic Measurements & Instrumentation

Question & Answers

Short persistence is needed for extremely high speed phenomena, to prevent smearing and interference caused when one image persists and overlaps with the next one. P11 phosphor is considered the best for photographing from the CRT screen.

3. Operating Voltages: the CRT requires a heater voltage of 6'3 volts ac or dc at 600mA. Several dc voltages are listed below. The voltages vary with the type of tube used. (i) Negative grid (control) voltage 14 V to - 200 V. (ii) Positive anode no. 1 (focusing anode) -100 V to - ll00 V (iii) Positive anode no. 2 (accelerating anode) 600 V to 6000 V (iv) Positive anode no. 3 (accelerating anode) 200 v to 20000 V in some cases

4. Deflection Voltages: Either ac or dc voltages will deflect the beam. The distance through which the spot moves on the screen is proportional to the dc, or peak ac amplitude. The deflection sensitivity of the tube is usually stated as the dc voltage (or peak ac voltage) required for each cm of deflection of the spot on the screen 5. Viewing Screen: The viewing screen is the glass face plate, the inside wall of which is coated with phosphor. The viewing screen is a rectangular screen having graticules marked on it. The standard size used nowadays is 8 cm x l0 cm (8 cm on the vertical and 10 cm on horizontal). Each centimeter on the graticule corresponds to one division (div). The standard phosphor color use d nowadays is blue

2. Explain about Triggered Sweep CRO? Ans: The continuous sweep is of limited use in displaying periodic signals of constant
frequency and amplitude. When attempting to display voice or music signals, the pattern falls in and out of sync as the frequency and amplitude of the music varies resulting in an unstable display.

A triggered sweep can display such signals, and those of short duration, e.g. narrow pulses. In triggered mode, the input signal is used to generate substantial pulses that trigger the sweep. Thus ensuring that the sweep is always in step with the signal that drives it. As shown in Fig. 2.1 GRIET/ECE

www.jntuworld.com

www.jntuworld.com

www.jwjobs.net

Electronic Measurements & Instrumentation

Question & Answers

resistance R3 and R 4 form a voltage divider such that the voltage Vo at the cathode of the diode is below the peak voltage i/o for UJT conduction. When the circuit is switched on, the UJT is in the non-conducting stage, and C, charges exponentially through RT. towards VBB until the diode becomes forward biased and conducts; the capacitor voltage never reaches the peak voltage required for UJT conduction but is clamped at Vo. If now a ve pulse of sufficient amplitude is applied to the base and the peak voltage Vp is momentarily lowered, the UJT fires. As a result, capacitor C. discharges rapidly through the UJT until the maintaining voltage of the UJT is reached; at this point the UJT switches off and capacitor CT charges towards VBB, until it is clamped again at VD fig 2.2 shows the output waveform

3. Explain briefly about delay line in triggered sweep circuit? Ans: Fig 3.1 indicates the amplitude of the signal raise time and the relative position of the
sweep generator output signal.

GRIET/ECE

www.jntuworld.com

www.jntuworld.com

www.jwjobs.net

Electronic Measurements & Instrumentation

Question & Answers

The diagram shows that when the delay line is not used, the initial part of the signal is lost and only part of the signal is displayed. To counteract this disadvantage the signal is not applied directly to the vertical plates but is passed through a delay line circuit, as shown in Fig.7.13. This gives time for the sweep to start at the horizontal plates before the signal has reached the vertical plates. The trigger pulse is picked off at a time t0 after the signal has passed through the main amplifier. The sweep generator delivers the sweep to the horizontal amplifier and the sweep starts at the HDP at time t0 + 80 ns. Hence the sweep starts well in time, since the signal arrives at the VDP at time t0 + 200 ns

GRIET/ECE

www.jntuworld.com

www.jntuworld.com

www.jwjobs.net

Electronic Measurements & Instrumentation

Question & Answers

4. Explain briefly about the Vertical amplifier and Horizontal deflecting system? Ans: The sensitivity (gain) and frequency bandwidth (B.W) response characteristics of the
oscilloscope are mainly determine by the vertical amplifier .Since the gain-B W. product is constant, to obtain a greater sensitivity the B.W. is narrowed, or vice-versa. Some oscilloscopes give two alternatives, switching to a wide bandwidth position, and switching to a high sensitivity position. Block Diagram of a Vertical Amplifier The block diagram of a vertical amplifier is a shown fig

4.1

The vertical amplifier consists of several stages, with fixed overall sensitivity gain expressed in V/div. The advantage of fixed gain is that the amplifier can be more easily designed to meet the requirements of stability and B.W. The vertical amplifier is kept within its signal handling capability by proper selection the input attenuator switch. The first element of the pre-amplifier is the input stage, often consisting of a FET source follower whose high input impedance isolates the amplifier from the attenuator. This FET input stage is followed by a BJT emitter follower, to match the medium impedance of FET output with the low impedance input of the phase inverter. This phase inverter provides two anti phase output signals which are required operate the pushpull output amplifier. The push-pull output stage delivers equal signal voltages of opposite polarity to the vertical plates of the CRT. The advantages of push-pull operation in CRO are similar to those obtained from push-pull operation in other applications; better voltage cancellation ran the source or power supply (i.e. dc), even harmonic suppression, especially large GRIET/ECE 5

www.jntuworld.com

www.jntuworld.com

www.jwjobs.net

Electronic Measurements & Instrumentation

Question & Answers

2nd harmonic is cancelled out, and greater power output per tube as a suit of even harmonic cancellation. In addition, a number of defocusing and non linear effects are reduced, because neither plate is at ground potential. HORIZONTAL DEFLETING SYSTEM: The horizontal deflecting system consist of a time base Generator and an output amplifier. Sweep or Time Base Generator A continuous sweep CRO using a UJT as a time base generator is shown in Fig. 7.8. The UJT is used to produce the sweep. When the power is first applied, the UJT is off and the CT charges exponentially through RT. The UJT emitter voltage VE rises towards VBB and when VE reaches the peak voltage VP, as shown in Fig. 4.3, the emitter to base '1' (B1) diode becomes forward biased and the UJT triggers ON. This provides a low resistance discharge path and the capacitor discharges rapidly. The emitter voltage VE reaches the minimum value rapidly and the UJT goes OFF. The capacitor recharges and the cycle repeats. To improve sweep linearity, two separate voltage supplies are used, a low voltage supply for UJT and a high voltage supply for the RTCT circuit.

RT is used for continuous control of frequency within a range and CT is varied or changed in steps for range changing. They are sometimes called as timing resistor and timing capacitor respectively. The sync pulse enables the sweep frequency to be exactly equal to the input signal frequency, so that the signal is locked on the screen and does not drift.

5. Explain the Basic principal involved in Signal Display Unit? Ans:The amplitude of a voltage may be directly measured on a calibrated viewing screen from
the length of the straight line trace it produces .This is entirely satisfactory for dc voltage. But the straight line tells little, or practically nothing, about the waveform of an ac voltage, pulsating voltage or transient. What is required is a graph of the voltage traced on the screen by the ac spot (a graph of amplitude versus time)

To obtain such a display the signal voltage is applied to the vertical plates (directly or through the vertical amplifier) and it moves the spot vertically I positions, corresponding to the instantaneous values of the signal. Simultaneous the spot is moved horizontally by a sweep voltage applied to the horizontal plates .The combined action of these two voltages causes the GRIET/ECE

www.jntuworld.com

www.jntuworld.com

www.jwjobs.net

Electronic Measurements & Instrumentation

Question & Answers

spot to produce a trace on the screen. The horizontal sweep voltage produces the time base by moving the spot horizontally with time, while the signal moves the spot vertically in proportional to the voltage at a particular instant of time.

There are two important sweep generator requirements: 1. The sweep must be linear (the sweep voltage must rise linearly to the maximum value required for full screen horizontal deflection of the spot). 2. The spot must move in one direction only, i.e. from left to right only, else the signal will be traced backwards during the return sweep. This means that the sweep voltage must drop suddenly after reaching its maximum value. These requirements call for a sweep voltage having a linear saw tooth waveform, as shown in Fig. 5.1.

Now at time t0, the sweep voltage is - E2, and the negative horizontal voltage moves the spot to point 1 on the screen. At this instant, the signal voltage is 0, so the spot rests at the left end of the zero line on the screen.

At time t1 the linearly increasing saw tooth reaches E1 which, being more positive than - E2, moves the spot to the screen, point 2. At this instant, the signal voltage is e, the +ve peak value, so the point represents its maximum upward deflection of the spot. At time t2, the saw tooth voltage is 0, there is no horizontal deflection and the spot is at the centre, point 3. At this instant the signal voltages is 0V.so, there is no vertical deflection either. At time t3, the saw tooth voltage is +E1 moving the spot to point 4.

At this instant, the signal is - e, the -ve peak value, so point 4 is the maximum downward deflection of the spot. At time t4, the saw tooth voltage is + E2, moving the spot to point 5. Now the signal voltage is 0, so the spot is not vertically reflected. Between t4 and t5, the saw tooth voltage falls quickly GRIET/ECE

www.jntuworld.com

www.jntuworld.com

www.jwjobs.net

Electronic Measurements & Instrumentation

Question & Answers

through 0 to its initial value of - E2, snapping the spot back to point 1, in time to sweep forward on the next cycle of signal voltage. When sweep and signal frequencies are equal, a single cycle appears on the screen, when the sweep is lower than the signal, several cycles appear (in the ratio of the two frequencies), and when sweep is higher than signal, less than one cycle appears. The display is stationary only when the two frequencies are either equal or integral multiples of each other. At other frequencies the display will drift horizontally. A saw tooth sweep voltage is generated by a multi vibrator, relaxation oscillator or pulse generator. The upper frequency generated by internal devices in the oscilloscope is 50-100 kHz in audio instruments, 500-1000 kHz in TV service instruments and up to several MHz in high quality laboratory instruments. In some oscilloscopes the sweep is calibrated in Hz or kHz, and in others it is calibrated in time units (us, ms, and s). The different types of sweep generated are as follows:

1. Recurrent Sweep: When the saw tooth, being an ac voltage alternates rapidly, re display occurs repetitively, so that a lasting image is seen by the eye. This -treated operation is recurrent sweep. 2. Single Sweep: The signal under study produces a trigger signal, which in turn produces a single sweep. 3. Driven Sweep: The saw tooth oscillator is a free running generator when p crated independently. There is a chance that the sweep cycle may start after the signal cycle, thereby missing a part of the signal. Driven sweep removes this possibility because it is fixed by the signal itself. The sweep and signal cycles start at the same time. 4. Triggered Sweep: In a recurrent mode, the pattern is repeated again and again. In this mode the voltage rises to a maximum and then suddenly falls to a minimum. Electron beam moves slowly from left to right, retraces rapidly to the left and I, pattern is repeated. The horizontal sweep action takes place whether the input signal is applied to the oscilloscope or not, and a horizontal line is displayed on the scope screen. A triggered sweep, on the other hand, does not start unless initiated by a trigger voltage, generally derived from an incoming signal. In the absence of the input signal the sweep is held off and the CRT screen is blanked. The continuous or recurrent sweep uses a free running multi vibrator (m/v) which covers a wide frequency range and can be locked into synchronization by input signal. Sync takes place when the sweep frequency and the input signal frequency are the same or when the former is a multiple of the latter.

A triggered scope does not use a continuous or recurrent sweep, but uses a mono stable multi vibrator which is in its off state until a trigger pulse arrives; hence there is no deflection on the screen. GRIET/ECE

www.jntuworld.com

www.jntuworld.com

www.jwjobs.net

Electronic Measurements & Instrumentation

Question & Answers

When an input signal is applied, a trigger pulse is generated and applied to the multi vibrator, which switches on and produces a sweep signal, and a trace appears on the screen. After a specific voltage, depending on the CRT beam arriving on the RHS, the multi vibrator switches back to its off state, causing the beam to return rapidly to the LHS. (The basic difference between recurrent and triggered scopes is that the recurrent sweep locks at the frequency of the input signal, while the triggered scope displays a trace for a specific period of time. Hence, the triggered scope is ON during a specific time interval and will display a waveform or a segment of waveform (e.g. a one shot waveform) regardless of the signal frequency. Hence transients or single clamped oscillations can be observed on the screen.) Most triggered scopes use a convenient feature of calibrating the sweep speed, in time per cm or division. Sweep frequency is the reciprocal of the time period.

5. Intensity Modulation In some applications an ac signal is applied to the control electrode of the CRT. This causes the intensity of the beam to vary in step with signal alternations. As a result, the trace is brightened during the +ve half cycles and diminished or darkened during ve half cycles. This process is called intensity modulation or Z-axis modulation (in contrast to Xaxis for horizontal and y-axis for vertical). It produces bright segments or dots on the trace in response to positive peak or dim segments or holes in response to negative peaks.

6. Explain the measurement of frequency using CRO? Ans: The frequency of a signal is measured using oscilloscope in two methods. They are,
1. Using calibrated oscilloscope 2. Using uncelebrated oscilloscope.

Measurement of Frequency using Calibrated Oscilloscope It is the indirect method of measurement of frequency. In this method, the frequency of unknown signal is measured by measuring its time period. Initially, the unknown frequency signal is applied to the vertical inputs of the CRO. Now the horizontal sweep is turned ON and the display appealing on the screen is adjusted by varying different control knobs provided on the front panel of CRO, till the signal is suitably displayed on the screen. After obtaining the display of good deflection, count the number of horizontal division for a complete cycle. From the counted horizontal divisions, the time period is computed as,

T=m*n
GRIET/ECE

www.jntuworld.com

www.jntuworld.com

www.jwjobs.net

Electronic Measurements & Instrumentation

Question & Answers

Where

m=Number of division in one complete cycle n=Setting of time base =Time/Division From the measured time period of the signal, the unknown frequency is calculated as,

f =1/T
2. Measurement of Frequency using Uncelebrated Oscilloscope? It is the direct method of measurement of frequency. In this method, initially a signal whose frequency is known is applied to the horizontal input terminals whereas a signal whose frequency is to be known is applied to the vertical input terminals of CRO. Now different control knobs provided on the CRO are adjusted till a pattern of loops appear on the CRT screen. Here, the displayed pattern of loops cut by horizontal line and also vertical line. The number of loops that are cut by horizontal line gives the frequency of signal applied to vertical plate (fv), whereas, the number of loops cut by vertical line gives the frequency of signal applied to horizontal plates (fh). Therefore we have. f v / fh = Number of loops cut by horizontal line/Number of loops cut by vertical line If the frequency of the signal applied to the horizontal input is 100 Hz. For the 100 Hz frequency signal, say the number of loops cut by horizontal line is 1 and the numbers of loops cut by vertical line are 2.

7. How is the vertical oscilloscope deflected? How does this differ from horizontal axis? Ans:
Deflection of the vertical axis of an oscilloscope is actually the deflection of electron beam in the vertical direction. To deflect the electron beam vertically, a pair of parallel plates is used. One vertical deflection plate (V1) is placed above the electron beam and the other vertical deflection plate is placed below the electron beam.

GRIET/ECE

10

www.jntuworld.com

www.jntuworld.com

www.jwjobs.net

Electronic Measurements & Instrumentation

Question & Answers

The vertical axis is deflected by applying vertical deflection voltages to the vertical deflection plates. The vertical deflection voltage is derived from the input signal to be analyzed by the oscilloscope. Usually, high Voltages are required to drive the deflection plates. Hence, a vertical amplifier is used to amplify the input signal to such an extent that it can deflect the electron beam.

The vertical amplifier amplifies the input signal and produces a two ended signal i.e., two signal voltages of equal amplitude but of opposite polarities. These two signal voltages are fed to the vertical deflection plates V1 and V2 (Y-plates) of CRO. The electron beam is a negatively charged beam because it contains electrons (negative charge particles). When the voltage at V1 is negative then the voltage at V2 will be positive. The positive deflection voltage at V1 attracts the electron beam to V2 and the negative voltage at V2 repels the electron beam from V1. As a result the electron beam is deflected vertically downwards (i.e., towards V2 plate). When the voltage at V1 is positive, then the voltage at V2 will be negative. The positive voltage at V1 attracts the beam to V1 and the negative voltage at V2 repels the beam from V2. As a result the electron beam is deflected vertically upwards (i.e., towards V1). The horizontal axis of an oscilloscope is deflected by applying horizontal deflection voltage to the horizontal deflection plates. The horizontal deflection voltage is derived from a saw-tooth signal. A pair of parallel plates (H1, and H2) placed vertically on either side of the electron beam in the CRT, serve as horizontal deflection plates (X-plates). One plate H1 is kept on left side and other plate R2 is kept on right side of the electron beam. A saw-tooth signal is amplified by the horizontal amplifier (identical to the vertical amplifier). The horizontal amplifier also produces two signal voltages of equal amplitude but of opposite polarities. These two voltages are fed to the horizontal deflection plates. When the deflection voltage at H1 is negative then the deflection voltage at H2 will be positive. The positive voltage at H2 attracts the electron beam to H2 and the negative voltage at H1 repels the electron beam from H1. As a result, the beam is deflected horizontally to the right. Similarly, the beam is deflected horizontally to the left when the voltage at H1 is positive and the voltage at H2 is negative. The difference in the deflection of vertical and horizontal axis lies in the voltages that drive the respective deflection plates. The vertical deflection plates are driven by the amplified input signal whereas the horizontal deflection plates are driven by an amplified saw-tooth signal which serves as a time-base signal. GRIET/ECE

11

www.jntuworld.com

www.jntuworld.com

www.jwjobs.net

Electronic Measurements & Instrumentation

Question & Answers

8. Mention the advantage of general purpose oscilloscope ?


ANS: Advantages of General Purpose Oscilloscope 1. It provides a graphical display of the amplitude of a signal as a function of time. Hence, it is used to measure various electrical parameters. 2. Amplitude of signals like voltage, current, power etc., can be measured by the oscilloscope. 3. Apart from amplitude measurement, it can measure frequency, phase angle, time delay of the signal, time between two events, and relative timing of two related signals. 4. It has an advantage over electro-mechanical measuring devices that it can respond very well to high frequency signals because it is completely an electronic device. 5. General purpose CRO's are used for maintenance of electronic equipment and laboratory work. 6. It can also be used to measure capacitance, inductance, etc.

9. Draw the Block diagram of a general purpose oscilloscope and explain its Basic operation ? Ans: Figure shows the basic block diagram of a general purpose oscilloscope.
A general purpose oscilloscope consists of following parts. 1. Cathode ray tube 2. Vertical amplifier 3. Delay line 4. Time base circuit 5. Horizontal amplifier 6. Trigger circuit 7. Power supply.

1. Cathode Ray Tube GRIET/ECE

12

www.jntuworld.com

www.jntuworld.com

www.jwjobs.net

Electronic Measurements & Instrumentation

Question & Answers

It is the heart of the oscilloscope. When the electrons emitted by the electron gun strikes the phosphor screen of the CRT, a visual signal is displayed on the CRT. 2. Vertical Amplifier The input signals are amplified by the vertical amplifier. Usually, the vertical amplifier is a wide band amplifier which passes the entire band of frequencies. 3. Delay Line As the name suggests that, this circuit is used to, delay the signal for a period of time in the vertical section of CRT. The input signal is not applied directly to the vertical plates because the part of the signal gets lost, when the delay Time not used. Therefore, the input signal is delayed by a period of time. 4. Time Base Circuit Time base circuit uses a uni junction transistor, which is used to produce the sweep. The saw tooth voltage produced by the time base circuit is required to deflect the beam in the horizontal section. The spot is deflected by the saw tooth voltage at a constant time dependent rate. 5. Horizontal Amplifier The saw tooth voltage produce by the time base circuit is amplified by the horizontal amplifier before it is applied to horizontal deflection plates 6. Trigger Circuit The signals which are used to activate the trigger circuit are converted to trigger pulses for the precision sweep operation whose amplitude is uniform. Hence input signal and the sweep frequency can be synchronized. 7. Power supply: The voltages require by CRT, horizontal amplifier and vertical amplifier are provided by the power supply block. Power supply block of oscilloscope is classified in to two types (1) Negative high voltage supply (2) Positive low voltage supply The voltages of negative high voltage supply is from -1000V to -1500V.The range of positive voltage supply is from 300V to 400V

10. What are the advantages of dual beam for multiple trace oscilloscopes? Ans:
GRIET/ECE

13

www.jntuworld.com

www.jntuworld.com

www.jwjobs.net

Electronic Measurements & Instrumentation Advantages of Dual Beam for Multiple Trace Oscilloscopes

Question & Answers

1. A multiple trace oscilloscope making use of dual beam provides a simultaneous display of the two input waveforms on the CRO screen. Hence dual beam CRO is used to compare one signal with another signal. 2. It can capture two fast transient events. 3. It also provides a continuous display of the signals, whereas the display of the two signals provided by a dual trace oscilloscope consists of small gaps in the trace. 4. It has two separate vertical channels for two input signals. 5. It can also have two separate time base circuits (i.e. horizontal deflection systems). Hence, in dual beam CRO two input signals can be swept horizontally at different rates. Due to this feature, a fast signal can be graphically compared with a slow signal simultaneously on the CRO screen.

GRIET/ECE

14

www.jntuworld.com

www.jntuworld.com Electronic Measurements & Instrumentation


www.jwjobs.net Question & Answers

UNIT-5
1: Explain about storage oscilloscope with block diagram?
Ans: Storage targets can be distinguished from standard phosphor targets by their ability to retain a waveform pattern for a long time, independent of phosphor persistence. Two storage techniques are used in oscilloscope CRTs, mesh storage and phosphor storage. A mesh-storage CRT uses a dielectric material deposited on a storage mesa as the storage target. This mesh is placed between the deflection plates and the standard phosphor target in the CRT. The writing beam, which is the focused electron beam of the standard CRT, charges the dielectric material positively e where hit. The storage target is then bombarded with low velocity electrons from a flood gun and the positively charged areas of the storage target allow these electrons to pass through to the standard phosphor target and thereby reproduced the stored image on the screen. Thus the mesh storage has both a storage target and a phosphor display target. The phosphor storage CRT uses a thin layer of phosphor to serve both as the storage and the display element. Mesh Storage It is used to display Very Low Frequencies (VLF) signals an: finds many applications in mechanical and biomedical fields. The convention-scope has a display with a phosphor persistence ranging from a few microseconds to a few seconds. The persistence can be increased to a few hours from a few seconds.

A mesh storage CRT, shown in Fig. 1.1, contains a dielectric material deposed on a storage mesh, a collector mesh, flood guns and a collimator, in addition BO all the elements of a standard CRT. The storage target, a thin deposition of a dielectric material such as Magnesium Fluoride on the storage mesh, makes use of a property known as secondary emission. The writing gun etches a positively charged pattern on the storage mesh or target by knocking off secondary emiss ion electrons. Because of the excellent insulating property of the Magnesium fluoride coating, this positively charged pattern remains exactly in the position where it is deposited. In order to make a pattern visible, a special electron gun, called the flood gun, is switched on (even after many hours). The electron paths ire adjusted by the collimator electrode, which constitutes a low voltage electrostatic lens system (to focus the electron beam), as shown in Fig. 1.2. Most of the electrons are stopped and collected by the collector mesh. Only electrons near the stored positive charge are pulled to the storage target with sufficient force to hit the phosphor screen. The CRT will now display the signal and it will remain visible as long as the flood guns operate. To erase the pattern on the storage mesh, a negative voltage is applied to neutralize the stored positive charge.

GRIET/ECE

www.jntuworld.com

www.jntuworld.com Electronic Measurements & Instrumentation


www.jwjobs.net Question & Answers

Since the storage mesh makes use of secondary emission, between the first and second crossover more electrons are emitted than are absorbed by the material, and hence a net positive charge results. Below the first crossover a net negative charge results, since the impinging electrons do not have sufficient energy to force an equal number to be emitted. In order to store a trace, assume that the storage surface is uniformly charge; and write gun (beam emission gun) will hit the storage target. Those areas of the storage surface hit by the deflecting beam lose electrons, which are collects by the collector mesh. Hence, the write beam deflection pattern is traced on the storage surface as a positive charge pattern. Since the insulation of the dielectric material is high enough to prevent any loss of charge for a considerable length of time, the pattern is stored. To view, the stored trace, a flood gun is used when the write gun is turned off. The flood gun, biased very near the storage mesh potential, emits a flood of electrons which move towards the collector mesh, since it is biased slightly more positive than the deflection region. The collimator ,a conductive coating on the CRT envelope with an applied potential, helps t o align the flood electrons so that they approach the storage target perpendicularly When the electrons penetrate beyond the collector mesh, they encounter either a positively charged region on the storage surface or a negatively charged region where no trace has been stored. The positively charged areas allow the electrons to pass through to the post accelerator region and the display target phosphor. The negatively charged region repels the flood electrons back to the collector mesh. Thus the charge pattern on the storage surface appears reproduced on the CRT display phosphor just as though it were being traced with a deflected beam.

2. Draw the block Diagram of a Dual Trace CRO and explain it? Ans:
Dual Trace Oscilloscope Figure 2.1 shows the block diagram of dual trace oscilloscope which consist of following steps, 1. Single electron gun 2. Two separate vertical input channels 3. Attenuators and pre-amplifiers 4. Electronic switch.

GRIET/ECE

www.jntuworld.com

www.jntuworld.com Electronic Measurements & Instrumentation


www.jwjobs.net Question & Answers

With the help of electronic switching the two separate input signals can be applied to single electron gun. Produces a dual trace display .Each separate vertical input channel uses separate attenuators and pre-amplifier stages, so, that the amplitude of each signal can be independently controlled. Output of the pre-amplifiers is given to the electronic switch, which passes one signal at a time into the main vertical amplifier of the oscilloscope. The time base-generator is similar to that of single input oscilloscope. By using switch S 2 the circuit can be triggered on either A or B channel, waveforms, or an external signal, or on line frequency. The horizontal amplifier can be fed from sweep generator or from channel B by switching S1. When switch S, is in channel B, its oscilloscope operates in the X-Y mode in which channel A acts as the vertical input signal and channel B as the horizontal input signal. From the front panel several operating modes can be selected for display, like channel B only, channel A only, channels B and A as two traces, and signals A + B, A - B, B ~ A or - (A + B) as a single trace. Two types of common operating mode are there for the electronic switch, namely, 1. Alternate mode 2. Chop mode.
1. Alternate Mode In case of alternate mode, electronic switch alternately connects the main vertical amplifier to channel A and B and adds a different D.C. component to each signal. Due to this D.C. component the beam alternately goes to the upper and lower half of the screen. Switching is carried out at the start of each new sweep of the time base generator. To obtain the cathode ray tube spot it traces the channel A signal on one sweep and the channel B signal on the succeeding sweep. The switching rate of the electronic switch is synchronized to the sweep rate. The alternate mode cannot be used for displaying very low frequency signals.

2. Chop Mode In chop mode electronic switch runs at a very high frequency of the order of 100-500 kHz. This results in connecting small segment of waveform A and waveform B , alternately to vertical amplifier to display it on the screen. The display will be a continuous line for each channel when chopping rate much faster than horizontal sweep rate. In case when the chopping rate is slow the continuity of the display is lost hence it is preferable to use alternate mode of operation.

GRIET/ECE

www.jntuworld.com

www.jntuworld.com Electronic Measurements & Instrumentation


www.jwjobs.net Question & Answers

3. Explain with Neat Block Diagram of Digital Storage oscilloscope? Ans:


Digital Storage Oscilloscope Storage cathode ray tube has several limitations. They are as follows, 1. There is a short duration of time, in which it can preserve a stored waveform, so the waveform may lose. 2. Trace of storage tube is not as fine as that of a normal CRT. 3. Writing rate of the storage tube is less than that of a conventional CRT which in turn limits the speed of the analog storage oscilloscope. 4. It is more expensive than a conventional CRT and requires additional power supplies. 5. Only one image can be stored. For comparing two traces they are to be superimposed on the same and displayed together. Digital storage oscilloscope is used to limit these limitations. In DSO, the waveform to be stored is digitized, stored in a digital memory and retrieved for display on the storage oscilloscope.

Stored waveform is continuously displayed by repeatedly scanning it. Therefore a conventional CRT can also be used for the display. The stored display can be displayed continuously as long as the power is applied to the memory which can be supplied from a small battery. Digitized waveform can be analyzed by oscilloscope or by reading the contents of the memory into the computer. Display of the stored data is possible in both amplitude versus time and x-y modes. In DSO, fast memory readout is used for CRT display in addition to this a slow readout is also possible which is used for development of hard copy externally. Figure shows the block diagram of DSO which consists of, 1. Data acquisition 2. Storage 3. Data display. Data acquisition is earned out with the help of both analog to digital and digital to analog converters, which is used for digitizing, storing and displaying analog waveforms. Overall operation is controlled by control circuit which is usually consists of microprocessor. Data acquisition portion of the system consist of a Sample-and-Hold (S/H) circuit and an analog to digital converter (ADC) which continuously samples and digitizes the input signal at a rate determined by the sample clock and transmit the digitized data to memory for storage. The control circuit determines whether the successive data points are stored in successive memory location or not, which is done by continuously updating the memories.

GRIET/ECE

www.jntuworld.com

www.jntuworld.com Electronic Measurements & Instrumentation


www.jwjobs.net Question & Answers

When the memory is full, the next data point from the ADC is stored in the first memory location writing over the old data. The data acquisition and the storage process is continues till the control circuit receive a trigger signal from either the input waveform or an external trigger source. When the triggering occurs, the system stops and enters into the display mode of operation in which all or some part of the memory data is repetitively displayed on the cathode ray tube. In display operation, two DACs are used which gives horizontal and vertical deflection voltage for the CRT Data from the memory gives the vertical deflection of the electron beam, while the time base counter gives the horizontal deflection in the form of staircase sweep signal. The screen display consist of discrete dots representing the various data points but the number of dot is very large as 1000 or more that they tend to blend together and appear to be a smooth continuous waveform. The display operation ends when the operator presses a front-panel button and commands the digital storage oscilloscope to begin a new data acquisition cycle.

4. Draw the simplified block diagram of the oscilloscope and explain in detail? Ans: SAMPLING OSCILLOSCOPE
An ordinary oscilloscope has a B.W. of 10 MHz the HF performance can be improved by means of sampling the input waveform and reconstructing its shape from the sample, i.e. the signal to be observed is sampled and after a few cycles sampling point is advanced and another sample is taken. The shape of the wave form is reconstructed by joining the sample levels together. The sampling frequency may be as low as 1/10th of the input signal frequency (if the input signal frequency is 100 MHz, the bandwidth of the CRO vertical amplifier can be as low as 10 MHz). As many as 1000 samples are used to reconstruct the original waveform.

Fig 4.1 shows a block diagram of a sampling oscilloscope. The input is applied to the sampling gate. The input waveform is sampled whenever a sampling pulse opens the sampling gate. The sampling must be synchronized with the input signal frequency. The signal is delayed in the vertical amplifier, allowing the horizontal sweep to be initiated by the input signal. The waveforms are shown in fig 4.2

GRIET/ECE

www.jntuworld.com

www.jntuworld.com Electronic Measurements & Instrumentation


www.jwjobs.net Question & Answers

At the beginning of each sampling cycle, the trigger pulse activates an oscillator and a linear ramp voltage is generated. This ramp voltage is applied to a voltage comparator which compares the ramp voltage to a staircase generate-When the two voltages are equal in amplitude, the staircase advances one step and a sampling pulse is generated, which opens the sampling gate for a sample of input voltage. The resolution of the final image depends upon the size of the steps of the staircase generator. The smaller the size of the steps the larger the number of samples and higher the resolution of the image

5. What are the difference between digital storage oscilloscope and conventional storage oscilloscope? Ans:

Conventional Storage Oscilloscope Digital Storage (Analog Storage Oscilloscope (ASO)) Oscilloscope (DSO) 1. It can store the given signal indefinitely as long 1. In this oscilloscope heavy amount-of power is as the small amount of power is supplied to the memory. 2. It always collects the data and stops when 2. It collects the data only after triggering. triggered. 3. It employs normal CRT, hence the cost of the 3. The cost of the tube is costlier than the storage tube is much cheaper than the storage tube used tube used in DSO. in ASO. 4. It cannot produce bright image for high 4. It can produce bright image even for high frequency signals. frequency signals. 5. In this oscilloscope, time base is generated, by 5. In this oscilloscope, time base is generated by a ramp circuit. a crystal clock. 6. 7. It has higher resolution than ASO. It has less operating speed than ASO. 6. It has lower resolution than DSO. 7. It has high operating speed than DSO. 8. It doesn't have aliasing effect. to be supplied to the storage CRT.

8. Because of aliasing effect the useful storage ' bandwidth is limited.

GRIET/ECE

www.jntuworld.com

www.jntuworld.com Electronic Measurements & Instrumentation


www.jwjobs.net Question & Answers

9. It cannot function under variable persistence 9. It can function under variable persistence storage mode. storage mode.

6. Explain the method of finding phase, frequency relation ship of two waveforms using Lissajous figures? Ans: Phase Measurement
When sinusoidal voltages simultaneously applied to vertical and horizontal plates, the pattern, appearing on the CRT is called as Lissajous pattern in this method the standard known frequency is applied to X-plate or horizontal plate. The resulting pattern appeared on the screen depends on phase relationship between the two frequencies. Equal voltages of same frequency but of different phase angles- cause the pattern to vary from a straight diagonal line to ellipses of different eccentricities. The sine of the phase angle between the voltages is given by equation, Sin = y1 /y2 = x1/x2 As shown in figure 6 . 1 the gains of horizontal or vertical amplifiers are adjusted so that ellipse fits exactly into a square marked by lines on the screen.

Frequency Measurement Lissajous pattern also helps to measure frequency. The signal whose frequency is to be measured is given I F-plates or vertical plates and the signal whose frequency is given to X-plates or horizontal plates. Now the know frequency or standard frequency is adjusted so Lissajous patterns can be obtained on the screen which depends on the ratio of two frequencies. In the given figure (2), Let, fv - Unknown frequency signal applied to vertical plates. fh - Known frequency signal applied to horizontal plates.

GRIET/ECE

www.jntuworld.com

www.jntuworld.com Electronic Measurements & Instrumentation


www.jwjobs.net Question & Answers

Two lines are drawn, one vertical and one horizontal so that they do not pass through any intersection o Lissajous pattern. Then the number of intersections of the horizontal and vertical lines with the Lissajous patterns and counted separately. So after finding the tangencies if we know we can easily calculate the unknown frequency applied to vertical plate. All electronic circuits in the oscilloscope like attenuators, time base generators, amplifiers cause some amount of time delay while transmitting signal voltage to deflection plates. We also know that horizontal signal is initiated or triggered by some portion of output signal applied to vertical plates of CRT. So the delay line is used to delay the signal for some time in the vertical section of CRT.

7. Explain the logic of a time base of a frequency counter? Ans:


The logic diagram of a time base for a frequency counter is shown in figure, the expected output from the time base are, (i) Reset pulse (ii) Gating pulse (iii) Store pulse.

GRIET/ECE

www.jntuworld.com

www.jntuworld.com Electronic Measurements & Instrumentation


www.jwjobs.net Question & Answers

The above pulses must be produced without overlap. If the wanted gate pulse duration is larger than the frequency period of the crystal then the frequency of crystal oscillator is divided by powers of ten. The binary counter consist of 1 6 states since it is a 4 bit counter The reset pulse is provided by the decoded zero state of the binary counter. To produce open gate pulse the 2nd state of the binary counter is decoded. To produce a delay period after the reset pulse the 1st state of the binary counter was not .used. The gate remains, open In between 2n state and 1 2 l ' state of the binary counter. Therefore, the close gate pulse is produced by the decoded 1 2 state of the binary counter. To produce a delay before storing the counter is latch during 14th state of the binary counter; the 1 3 ' state i not decoded The non overlap between the reset and store pulses can provided-by the non decoded 15th state of the binary counter. A switch is used i n this arrangement, to select the, gate time intervals of the frequency counter. In the above figure 1 sec, 0.1 sec, 0.01 sec are the available gate time intervals and 1 Hz, 1 0 Hz, 100 Hz are the available input frequencies

8. Explain in detail about various types of attenuators? Ans:


ATTENUATORS Attenuators are designed to change the magnitude of the input signal seen at input stage, while presenting constant impedance on all ranges at the attenuator input. A compensated RC attenuator is required to attenuate all frequencies equal without this compensation, HF signal measurements would always have to take the input circuit RC time constant into account. The input attenuator must provide the correct 1 -2-5 sequence while maintaining a constant input impedance, as well as maintain both the input impedance a: attenuation over the frequency range for which the oscilloscope is designed. Uncompensated Attenuators The circuit diagram shown in Fig. 8.1 gives a resistive divider attenuator connected to an amplifier with a 10 pf input capacitance. If the input impedance of the amplifier is high, the input impedance of the attenuator is relatively constant immaterial of the switch setting of the attenuator. The input impedance, as seen by the amplifier, changes greatly depending on the setting of the attenuator. Because of this, the RC time constant and frequency response of the amplifier are dependent on the setting of the attenuator, which an undesirable feature.

Simple Compensated Attenuator The diagram in Fig.8.1 shows an attenuator with both resistive and capacitive voltage dividers. The capacitive voltage dividers improve the HF response of the attenuator. This combination of capacitive and resistive voltage dividers is known as a compensated attenuator. For oscilloscopes where the frequency range extends to 100 MHz and beyond, more complex dividers are used.

GRIET/ECE

www.jntuworld.com

www.jntuworld.com

www.jwjobs.net

Fig 8.2 shows an attenuator divider between the input and output of the vertical deflection preamplifier. The input attenuator provides switching powers of 10, while attenuator at the output of the vertical preamplifier provides 1-2-5 attenuation. Practically all oscilloscopes provide a switchable input coupling capacitor, as shown in Fig. 8.2 The input impedance of an oscilloscope is 1 M which is shunted with an input capacitance of 10-30 pf. If a probe were connected to the oscilloscope, the input impedance at the probe tip would have a greater capacitance because of the added capacitance of the probe assembly and of the connecting shielded cable. If it is desired for HF oscilloscopes to have an input capacitance of much less than 20-30 pf, an attenuator probe is used. Figure 8.1 shows a 10 to 1 attenuator probe connected to the input of the oscilloscope. Within the probe tip is a 9M resistor and shunted across this resistor is a capacitor .This capacitor is adjusted so that the ratio of the shunt capacitance to the series capacitance is exactly 10 to 1. The attenuator probe often called a 10 to 1 probe provides an approximately 10 to 1 reduction in the input capacitance. However, it is also gives a 10 to 1 reduction in overall oscilloscope sensitivity. The input capacitance is not constant from one oscilloscope to another hence the probe is provided with an adjustable compensating capacitor .If the ratio of the series to shunt is not adjusted precisely to 10 to 1; the frequency response of the oscilloscope will be flat.

GRIET/ECE

10

www.jntuworld.com

www.jntuworld.com

www.jwjobs.net

9. Explain in detail about Digital read out oscilloscope? Ans: The digital read out oscilloscope instrument has a CRT display and a counter display. The
diagram shown is of an instrument where the counter measures the time (Fig. 9.1)

The input waveform is sampled and the sampling circuit advances the sampling position in fixed increments, a process called strobing. The equivalent me between each sample depends on the numbers of sample taken per cm and the sweep time/cm, e.g. a sweep rate of 1 nano-sec/cm and a sampling rate of J samples/cm gives a time of 10 pico-sec/sample. Figure 9.1 shows a block diagram of a digital read out oscilloscope when measuring voltage. Two intensified portions of the CRT trace identify 0% and 100% zones position. Each zone can be shifted to any part of the display. The voltage divider Bps between the 0% and 100% memory voltage GRIET/ECE

11

www.jntuworld.com

www.jntuworld.com

www.jwjobs.net

are set for start and stop timing, The coincidence of any of the input waveforms with the selected percentage point is sensed by this voltage comparator. The numbers of the clock pulse which correspond to the actual sample taken are read out digitally in a Nixie display tube in ns, s, ms or seconds. Figure 9.2 shows a block diagram of a digital readout CRO when used for voltage to time conversion. The CRT display is obtained by sampling the 0% reference voltage as chosen by the memory circuit. A linear ramp generator produces a voltage; when the ramp voltage equals the 0% reference the gate opens. When the ramp equals 100% reference the gate closes. The number of clock pulses that activate the counter is directly proportional to the voltage between the selected references and is read out in mV or volts by the Nixie tube display.

10. Discuss in detail about (i) A and B chopped (ii) A and B alternate? Ans:
(i) A and B Chopped Display Mode In chopped display mode of dual-trace oscilloscope, the electronic switch switches between the channels A and B at very high frequency (usually at 100-500 kHz). Due to very fast switching between the input channels, small segments (of about 1 us duration) of each input waveform get coupled alternately to the vertical amplifier and get traced on the screen. Thus, the display of input signal waveform will be a dashed-line trace of the actual input waveform, with small gaps between its segments. The continuity of the trace of the waveform depends on chopping (i.e., switching) frequency and the frequency of the input signal. Usually, the chopping frequency must be higher than the input signal frequency and sweep frequency. In chop mode, the traces of high frequency signals do not appear continuously as shown in fig 10.1. Whereas in case of low frequency signals, the gaps between the GRIET/ECE

12

www.jntuworld.com

www.jntuworld.com

www.jwjobs.net

segments are so small that the trace of low frequency signal appears to be a continuous waveform. Hence, chop mode display is usually preferred for low frequency signals.

(ii) A and B Alternate Display Mode The alternate display mode also involves alternate switching of the input signals to the vertical amplifier with the help of an electronic switch but at low switching rates. In this mode, the electronic switch is allowed to switch between the input channels at a frequency equal to the sweep frequency. Due to this, the signal on channel A gets connected to the vertical amplifier and is traced on the CRO screen during one sweep. Similarly, the signal on channel B gets traced during the succeeding sweep, as shown in fig 10.2. As the two input waveforms are traced at different times, they do not appear to be displayed simultaneously on the screen.

The alternate mode is preferred for high frequency signals (rather than low frequency signals) because the waveform traces appear to be continuous and displayed simultaneously due to high repetition frequency. GRIET/ECE

13

www.jntuworld.com

www.jntuworld.com Electronic Measurements & Instrumentation

www.jwjobs.net Question & Answers

UNIT -6 1. Draw the Maxwells Bridge Circuit and derives the expression for the unknown element at balance? Ans: Maxwell's bridge, shown in Fig. 1.1, measures an unknown inductance in of standard arm offers the advantage of compactness and easy shielding. The capacitor is almost a loss-less component. One arm has a resistance R x in parallel with Cu and hence it is easier to write the balance equation using the admittance of arm 1 instead of the impedance. The general equation for bridge balance is

From equation of Zx we get

Equating real terms and imaginary terms we have

Also

Maxwell's bridge is limited to the measurement of low Q values (1 -10).The measurement is independent of the excitation frequency. The scale of the resistance can be calibrated to read inductance directly. The Maxwell bridge using a fixed capacitor has the disadvantage that there an interaction between the resistance and reactance balances. This can be avoids: by varying the capacitances, instead of R2 and ft, to obtain a reactance balance. However, the bridge can be made to read directly in Q . The bridge is particularly suited for inductances measurements, since comparison on with a capacitor is more ideal than with another inductance. Commercial bridges measure from 1 1000H. With 2% error. (If the Q is very becomes excessively large and it is impractical to obtain a satisfactory variable standard resistance in the range of values required).

2. Draw the Wiens Bridge Circuit and derives the expression for the unknown element
balance?

at

GRIET/ECE

www.jntuworld.com

www.jntuworld.com Electronic Measurements & Instrumentation

www.jwjobs.net Question & Answers

Ans: Wien Bridge shown in Fig. 2.1 has a series RC combination in one and a parallel combination in the adjoining arm. Wien's bridge in its basic form is designed to measure f r e q u ency. It can also be used for the instrument of an unknown capacitor with great accuracy,

The impedance of one arm is

The admittance of the parallelarm is

Using the bridge balance equation,we have We have

Therefore

Equating the real and imaginary terms we have

Therefore

(1.1) And

GRIET/ECE

www.jntuworld.com

www.jntuworld.com Electronic Measurements & Instrumentation

www.jwjobs.net Question & Answers

The two conditions for bridge balance, (1.1) and (1.3), result in an expression determining the required resistance ratio R2/R4 and another express determining the frequency of the applied voltage. If we satisfy Eq. (1.1) an also excite the bridge with the frequency of Eq. (1.3), the bridge will be balanced. In most Wien bridge circuits, the components are chosen such that R 1 = R3 = R and C1 = C3 = C. Equation (1.1) therefore reduces to R2IR4 =2 at Eq. (1.3) to f= 1/2RC, which is the general equation for the frequency of fl bridge circuit. The bridge is used for measuring frequency in the audio range. Resistances R1 and R3 can be ganged together to have identical values. Capacitors C1 and C3 are normally of fixed values The audio range is normally divided into 20 - 200 - 2 k - 20 kHz range In this case, the resistances can be used for range changing and capacitors, and C3 for fine frequency control within the range. The bridge can also be use for measuring capacitances. In that case, the frequency of operation must be known. The bridge is also used in a harmonic distortion analyzer, as a Notch filter, an in audio frequency and radio frequency oscillators as a frequency determine element. An accuracy of 0.5% - 1% can be readily obtained using this bridge. Because it is frequency sensitive, it is difficult to balance unless the waveform of the applied voltage is purely sinusoidal.

3. Draw the Hays Bridge Circuit and derives the


expression for the unknown element at balance? Ans: The Hay Bridge, shown in Fig. 3.1 differs from Maxwell's bridge by having a resistance R1 in series with a standard capacitor C1 instead of a parallel. For large phase angles, R1 needs to be low; therefore, this bridge is more convenient for measuring high Q coils. For Q=10, the error is 1 %, and for Q = 30, the is 0.1%. Hence Hay's bridge is preferred for coils with a high Q, and Ma bridge for coils with a low Q. At balance

GRIET/ECE

www.jntuworld.com

www.jntuworld.com Electronic Measurements & Instrumentation

www.jwjobs.net Question & Answers

Substiting these values in the balance euaqation we get

Equating the real and imaginary terms we have

inand

Solving for Lx and Rx we have , Substituting for Rx in equation 1.1

Multiply both sides by C1 we get

Therefore,

Substituing for Lx in eq 1.2 we get

\ The term appears in the expression for both LX and RX. This indicates that the bridge is frequency sensitive. Hay Bridge is also used in the measurement of incremental inductance. The inductance balance equation depends on the losses of the inductor (or Q) and also on the operating frequency. AN inconvenient feature of this bridge is that the equation giving the balance condition for inductance, contains the multiplier 1/ (1 + 1/Q2). The inductance balance thus depends on its Q and frequency. GRIET/ECE

www.jntuworld.com

www.jntuworld.com Electronic Measurements & Instrumentation

www.jwjobs.net Question & Answers

Therfore

For a value of Q greater than 10, the term L/Q2 will be smaller than 1/100 and can be therefore neglected. Therefore LX = R2 R3 C1 which is the same as Maxwell's equation. But for inductors with a Q less than 10, the 1/Q2 term cannot be neglected. Hence this bridge is not suited for measurements of coils having Q less than 10. A commercial bridge measure from 1H - 100 H with 2% error.

4. Draw the Wheat stones Bridge Circuit and derives the expression for the unknown element at
balance? Ans: WHEATSTONE'S BRIDGE (MEASUREMENT OF RESISTANCE) Whetstones bridge is the most accurate method available for measuring resistances and is popular for laboratory use. The circuit diagram of a Wheatstone bridge is given in Fig. 11.1. The source of emf and switch is connected to points A and S, while a sensitive current indicating meter, the galvanometer, is connected to points C and D. The galvanometer is a sensitive micro ammeter a zero center scale. When there is no current through the meter, the galvanometer pointer rests at 0, i.e. mid scale. Current in one direction causes the points deflect on one side and current in the opposite direction to the other side.

When SW1 is closed, current flows and divides into the two arms at point A i.e. I1 and I2. The bridge is balanced when there is no current through the galvanometer, or when the potential difference at points C and D is equal, i.e. the potential across the galvanometer is zero. 4.1 To obtain the bridge balance equation, we have from the fig

For the galvanometer current to be zero, the following conditions should be satisfied

GRIET/ECE

www.jntuworld.com

www.jntuworld.com Electronic Measurements & Instrumentation

www.jwjobs.net Question & Answers

This is the equation for the bridge to be balanced. In a practical Whetstones bridge, at least one of the resistance is made adjustable, to permit balancing. When the bridge is balanced, the unknown resistance (normally connected at R4) may be determined from the setting of the adjustable resistor, which is called a standard resistor because it is a precision device having very small tolerance.

Hence

5. Explain about Ac bridges and also the precautions to be taken while using a Bridge?
Impedances at AF or RF are commonly determined by means of an ac Wheatstone bridge. The diagram of an ac bridge is given in Fig. 11.17. This bridge is similar to a dc bridge, except that the bridge arms are impedances. The bridge is excited by an ac source rather than dc and the galvanometer is replaced by a detector, such as a pair of headphones, for detecting ac. When the bridge is balanced,

Ans:

Where Z1, Z2, Z3 and Z4 are the impedances of the arms,

and are vector complex quantities that possess phase angles. It is thus necessary to adjust both the magnitude and phase angles of the impedance arms to achieve balance, i.e. the bridge must be balanced for both the reactance and the resistive component.

PRECAUTIONS TO BETAKEN WHEN USING A BRIDGE Assuming that a suitable method of measurement has been selected and that i.e. source and detector are given, there are some precautions which must be observed to obtain accurate readings. The leads should be carefully laid out in such a way that no loops or long lengths enclosing magnetic flux are produced, with consequent stray inductance errors. With a large L, the self-capacitance of the leads is more important than there inductance, so they should be spaced relatively far apart. In measuring a capacitor, it is important to keep the lead capacitance as low as possible. For this reason the leads should not be too close together and should be made of fine wire. In very precise inductive and capacitances measurements, leads are encased in metal tubes to shield them from mutual electromagnetic action, and are used or designed to completely shield the bridge

GRIET/ECE

www.jntuworld.com

www.jntuworld.com Electronic Measurements & Instrumentation

www.jwjobs.net Question & Answers

6. Draw the Andersons Bridge Circuit and derives the expression for the unknown element at
balance? Ans : The Anderson Bridge is a very important and useful modification of the Maxwell-Wein bridge as shown in the fig 6.1 (a)

The balance condition for this bridge can be easily obtained by converting the mesh impedance C,R3,R5 to a equivalent star with the star point 0 as shown in fig 6.1 (b) by using star/delta transformation As per delta to star transformation

Hence with reference to fig 6.1(b) it can be seen that

For balance bridge condition,

Therefore,

Simplifying, GRIET/ECE

www.jntuworld.com

www.jntuworld.com Electronic Measurements & Instrumentation

www.jwjobs.net Question & Answers

Equating the real terms and imaginary terms

And therefore

This method is capable of precise measurement of inductance and a wide range of values from a few H to several Henry.

7. Compare the measuring accuracy of a Wheat stone bridge with the accuracy of an ordinary meter? Ans:
Errors in PMMC Ammeters The accuracy in a moving coil instruments example, an ammeter is dictated by the following sources of errors. (i) Weakening of permanent magnets due to ageing and temperature effects. (ii) The weakening of springs due to regular usage and temperature effects.

GRIET/ECE

www.jntuworld.com

www.jntuworld.com Electronic Measurements & Instrumentation

www.jwjobs.net Question & Answers

(iii) Variation in Resistance of moving coil with temperature. The copper wire wound has a temperature coefficient of about 0.004/C. When this instrument used for measurement of very small currents in the milli-amp or micro-amp range, the moving coil is directly connected to the output terminals of the instrument. The indication would then decrease by 0.04% per C rise in temperature for a constant current. Errors in Wheatstone bridge Measurements The accuracy of measurement of resistance in a Wheatstone bridge is affected by the following sources. (i) Resistance of connecting leads and contact resistance.

(ii) Thermo electric effects. The galvanometer deflection is affected by thermo electric emfs which are often present in the measuring circuit. (iii) Temperature effects: The change in resistance due to variation of temperature causes serious errors in measurement. The error are more predominant in the case of resistors are made up of materials having high temperature coefficients. In the case of copper having a temperature coefficient of 0.004/C, a change in temperature of 1C causes an error of about 0.4%.

8. Draw the circuit of a basic Q-meter diagram and explain its principal of operation using a vector diagram? Ans: The instrument which measures some of the electrical properties of coils and capacitors is referred as Q-meter. The working principle of a Q-meter depends on the characteristics of a series resonance circuits, i.e., the voltage drop across the coil or capacitors is equal to the applied voltage times the Q factor of the circuit. Thus if the circuit subjected to a fixed voltage, the voltmeter connected across the capacitor is calibrated to indicate the Q value directly. A series resonance circuit and its voltage and current relationship at resonance conditions are illustrated in figure 8.1 (i) and fig 8.1 (ii) respectively.

At resonance condition, XL = X C E C = I XL=I X C E=IR

Where, GRIET/ECE

www.jntuworld.com

www.jntuworld.com Electronic Measurements & Instrumentation

www.jwjobs.net Question & Answers

X C = Capacitive reactance XL=Inductive reactance I = Current flowing through the circuit E = Applied voltage R = Resistance of the coil The Q factor or the magnification of the circuit is defined as,

From the above equation it is clear that if the voltage E is maintained at a fixed level, the voltmeter across the capacitor can be calibrated in terms of Q directly. The circuit arrangement of basic and practical Q-meter is shown below.

The oscillator is a wide range RF oscillator that supplies the oscillations whose frequency lies between 50 kHz to 50 MHz and delivers current to Rsh which is a shunt resistance of low value, and is typically around 0.02Q. Therefore the Rsh introduces very negligible (almost no resistance) resistance into the oscillator circuit. Thus it represents a voltage source of magnitude E with a very low internal resistance. The voltage across Rsh is measured using a thermocouple meter that is marked as 'multiply Q by meter. The voltage drop across the tuning capacitor or resonating capacitor EC is measured by means of an

electronic voltmeter. The scale of this electronic voltmeter is calibrated in terms of Q values directly. To carry out the measurement the unknown components is connected across the test terminals and the circuit is adjusted to resonance using any one of the two methods given below. By setting the frequency of the oscillator to a certain given value and adjusting the tuning capacitor.

GRIET/ECE

10

www.jntuworld.com

www.jntuworld.com Electronic Measurements & Instrumentation

www.jwjobs.net Question & Answers

By presetting the capacitor to a required value and varying the frequency of oscillator. The Q value indicate on the output meter should be multiplied by the index setting of the 'multiply Q by meter to get actual, or accurate Q value. The indicated value of Q on the output meter is known as 'circuit Q' since it includes the losses of voltmeter tuning capacitor and insertion resistor. The effective Q value of the measured coil will be higher than indicated Q or circuit Q. This difference is small therefore it can be neglected. However this difference is large if the resistance of the coil is small compared to the insertion resistor value. The inductance of the coil can be found from the known values of c (resonating capacitance) and / (frequency). Since XC =XL 2fL= 1/2fC Therefore L = 1 / ( 2 f )
2

C Henry

9 What are the applications of wheat stone bridge and explain its limitations? Ans:
Applications of Wheatstone bridge 1. The basic application of a Wheatstone bridge is measurement of resistance. It is used to measure medium resistance values. 2. It can also be used to measure inductance and capacitance values. 3. Various industrial applications involve measurement of physical quantities (such as temperature, pressure, displacement etc) in terms of electrical resistance. The various industrial applications in which a Wheatstone bridge is used are. (i) (ii) (iii) (iv) (v) (vi) 4. Temperature measurement systems involving electrical resistance thermometers as temperature sensors. Pressure measurement systems involving strain gauge as secondary transducer. Measurement of static and dynamic strains. It is used with explosive meter to measure the amount of combustible gases in a sample. Temperature measurement systems involving electrical resistance thermometers as temperature sensors. Pressure measurement systems involving strain gauge as secondary transducer. Measurement of static and dynamic strains.

5. It is used with explosive meter to measure the amount of combustible gases in a sample.

Limitations of Wheatstone bridge 1. Wheatstone bridge is not suitable for measuring low resistances because the resistance of leads and contacts of the bridge cause errors in the value measured by the Wheatstone bridge and thus affects the measurement of low resistances. 2. Wheatstone bridge cannot be used for measurement of high resistance also, because a galvanometer is not sensitive to the imbalance of the bridge caused by the high resistance of the bridge. This problem can be overcome by replacing the galvanometer with a Vacuum Type Volt Meter (VTVM) and by replacing the battery with a power supply. GRIET/ECE

11

www.jntuworld.com

www.jntuworld.com Electronic Measurements & Instrumentation

www.jwjobs.net Question & Answers

3. A Wheatstone bridge cannot be used in high temperature or temperature-varying environment because the resistance of the arms of the bridge changes due to change in temperature. 4. The resistance of the bridge arms also changes due to heating effect of the current passing through the resistance. Flow of very large current through the resistors leads to a permanent change of resistance value .

10. Explain the FM recording method? Ans:


FM Recording In FM (Frequency Modulation) recording technique the principle for recording an input signal is as follows, A high frequency carrier signal is frequency modulated by the input signal (i.e. the signal to be recorded) and then this frequency modulated carrier signal is recorded and reproduced by the basic magnetic recording procedure. The input signal is extracted from the reproduced modulated waveform by processing it through a demodulator and low pass filter. The frequency with which the carrier signal oscillates is known as centre frequency (fc). A frequency modulation recording system consists of a frequency modulator, frequency demodulator, a low pass filter and the basic magnetic recording circuit as shown in the figure (1) below.

When the input signal (to be recorded) is applied to the frequency modulator, the frequency of the carrier signal gets modulated as per the frequency level of the input signal. When there is no input (i.e. input voltage = 0) the centre frequency of the carrier signal remains unchanged and hence the output of modulator oscillator at a frequency When the input signal is applied and if the input voltage is positive, the carrier frequency gets deviated by certain percentage in one direction. If the input voltage is negative the carrier frequency gets deviated by certain percentage in the opposite direction. A positive voltage input signal increases the carrier frequency fc, while a negative voltage input signal decreases the carrier frequency, as shown in the figure (2) below. GRIET/ECE

12

www.jntuworld.com

www.jntuworld.com Electronic Measurements & Instrumentation

www.jwjobs.net Question & Answers

For A.C input signals, the output of the modulator will be a signal of varying frequency, and the variation in frequency is directly proportional to the amplitude (voltage) of the input signal. The output of the frequency modulator is then fed to the recording head of the system. The recording head records this modulated signal on the magnetic tape. Later, when the tape is passed through the reproduce head, it produces a voltage which represents the same modulated waveform being recorded on the tape. The output of the reproduce head is then demodulated by passing it through a frequency demodulator. Hence, the output of the demodulator consists of carrier frequency and other unwanted frequencies along with frequency of input signal. In order to remove these unwanted frequencies the output of the demodulator is given to low pass filter, which allows the frequency components of only input signal to pass through it. Thus, in this way a signal is recorded and reproduced by FM recording technique.

Advantages of FM Recording 1. In FM recording technique, the D.C component of the signal being recorded is preserved.

2. In this recording technique, the recorded signal is accurately reproduced because this technique is independent of amplitude variations. 3. It is widely employed for recording the output voltages of transducers such as force, pressure or acceleration transducers. 4. It is also used in instrumentation systems for multiplexing purposes.

5. FM recorders can record a wide range of frequency signals i.e. D.C signals to several kHz signals.

GRIET/ECE

13

www.jntuworld.com

www.jntuworld.com Electronic Measurements & Instrumentation

www.jwjobs.net Question & Answers

Disadvantages of FM Recording 1. FM recording systems are more complex than direct recording systems because FM recorders utilize modulation and demodulation devices. FM recording is very sensitive to fluctuations in speed of tape. The variations in tape speed leads to unnecessary modulation of the carrier signal. For efficient recording, the FM recording technique requires the tape speed to be high and constant. FM recorders are costlier than direct recording systems because they require a high quality of tape transport and speed control. The frequency response of FM recorders is limited to 80 kHz.

2.

3.

4. 5.

GRIET/ECE

14

www.jntuworld.com

www.jntuworld.com

www.jwjobs.net

Electronic Measurements & Instrumentation

Question & Answers

UNIT 7
1. What is transducer? Write the classifications of transducers? Ans:
Transducer A measuring device which measures and converts nonelectrical variable into electrical variable is known as transducer. Transducers are classified into several types. However, these can be categorized into five types. They are, 1. 2. 3. 4. 5. Classification on the basis of transduction principle used. Active and passive transducers Analog and digital transducers Primary and secondary transducers Transducers and inverse transducers.

1. Classification on the Basis of Transduction Principle Used This classification is done depending on the transduction principle i.e., how the input variable is being converted into capacitance, resistance and inductance values. (These are named as capacitive transducer, resistive transducer and inductive transducer respectively).

Examples of Capacitive Transducer 1. Dielectric gauge.

Applications 1.It is used to measure, (i) Thickness and (ii) Liquid level. 2.It is used to measure, (i) Noise (ii) Speech and Music

2. Capacitor Microphone.

GRIET/ECE

www.jntuworld.com

www.jntuworld.com

www.jwjobs.net

Electronic Measurements & Instrumentation

Question & Answers

Examples of Resistive Transducer 1. Resistance thermometer.

Applications 1. Used in the measurement or, (i) Temperature and (ii) Radiant heat.

2. Potentiometer device.

2. (i) Used in displacement measurement and (ii) Used in pressure measurement. Applications 1. It is used to measure, (i) Pressure (ii) Vibrations (iii) Position and ( iv) Displacement. 2. It is used to measure, (i) Sound (ii) Force (iii) Pressure.

Examples of Inductive Transducer 1. Reluctance pick up.

2. Magnetostriction gauge.

2.

Active and Passive Transducers

Active Transducer The transducer which does not requires any external excitation to provide their outputs are referred as active transducer. Examples of Active Transducer 1. Photo voltaic cell. 2. Thermocouple. Applications 1. (i) Used in light meters (ii) Used in solar cells. 2. Used to measure, (i) Temperature (ii) Radiation and (iii) Heat flow. 2

GRIET/ECE

www.jntuworld.com

www.jntuworld.com

www.jwjobs.net

Electronic Measurements & Instrumentation

Question & Answers

Passive Transducer The transducer which requires an external excitation to provide their output is referred as passive transducer. Examples of Passive Transducer 1. Capacitive transducers. 2. Resistive transducers. 3. Inductive transducers. Applications 1. Used to measure liquid level, noise, thickness etc. 2. Used to measure temperature, pressure, displacement etc. 3.Used to measure pressure, vibration, position, displacement etc.

3. Analog and Digital Transducers Analog Transducer The transducer which produces their outputs in analog form or a form which is a continuous function of time is referred as analog transducer. Examples of Analog Transducer 1.Strain gauge Applications 1. Used to measure, (i) Displacement (ii) Force and (iii) Torque. 2.Used to measure, (i) Temperature and (ii) Flow.

2. Thermistor

Digital Transducer The transducer which produces their outputs in digital form or a form of pulses is referred as digital transducers. Examples of Digital Transducer Turbine meter GRIET/ECE

Applications Used in flow measurement. 3

www.jntuworld.com

www.jntuworld.com

www.jwjobs.net

Electronic Measurements & Instrumentation

Question & Answers

4. Primary and Secondary Transducers Primary Transducer The transducer which sends the measurement and converts them into another variables (like displacement, strain etc.) and whose output forms the input of another transducer is called as primary transducer. Examples of Primary Transducer 1. Bourdon tube 2. Strain gauge Applications 1. Used in pressure 2. Used in measurements

Secondary Transducer The transducer which converts the output of first transducer into an electrical output called secondary transducer. Examples of Secondary Transducer LVDT Applications Used to measure, (i) Displacement J (ii) Force (iii) Pressure and (iv) Position

5. Transducers and Inverse Transducers Transducers A measuring device which measures and converts nonelectrical variable into electrical variable is known as transducer. Example of Transducer Thermocouple Applications Used to measure, j (i) Temperature (ii) Radiation and (iii) Heat flow 4

GRIET/ECE

www.jntuworld.com

www.jntuworld.com

www.jwjobs.net

Electronic Measurements & Instrumentation

Question & Answers

Inverse Transducer A measuring device which measures and converts an electrical variable into nonelectrical variable is known as inverse transducer. Example of Inverse Transducers Piezo-electric crystal Applications Used to measure, (i) Pressure (ii) Vibration and acceleration

2. What parameters should be considered in selecting a transducer? Ans:


Parameters to be considered in the selection of a transducer for a particular application are. 1. Operating Principle Basically the transducers are selected based on their operating principle. Examples of operating principles used by the transducers are resistive, capacitive, piezoelectric, inductive, up to electronic principle etc. Operating Range

2.

This factor is considered so that the transducer should be able to function within the specified range with good resolution. Every transducer should be provided with some rating within which there will be breakdown in its function. 3. Accuracy It is one of the most desired characteristic of any transducer. If the transducer doesn't needs frequent calibration, it must have high degree of accuracy and repeatability. Because errors may occur due to the sensitivity of the transducer to other stimulations. 4. Sensitivity It is also a desired characteristic of a transducer. Every transducer should be sufficiently sensitive to provide some output that can be sufficient and detectable. 5. Stability and Reliability The transducer should have high degree of stability during its function and also storage life. It should also have a high degree of reliability. Usage and Ruggedness 5

6.

GRIET/ECE

www.jntuworld.com

www.jntuworld.com

www.jwjobs.net

Electronic Measurements & Instrumentation

Question & Answers

The ruggedness, size and weight of a transducer should be chosen depending on the application in which it is used. 7. Transient Response and Frequency Response The transducer should have required time domain specifications such as, settling time, rise time, peak over shoot and small dynamic error etc. Loading Effects The transducers should undergo minimum loading effect so that if can provide accurate measurement. The parameters of a transducer are that, which characterize the loading effect is its input and output impedances.lt is considered in order to get minimum loading effects (Which can be neglected). For minimum loading effect the transducer should have low output impedance and high input impedance. 9. Electrical Parameters The type and length of cable required, signal to noise ratio in case the transducer is used with amplifiers and frequency response limitations should also be considered. 10. 11. 12. Ability to be insensitive to unwanted signals (or the ability to be sensitive to desired signals). Environmental compatibility. Static Characteristics The selected transducer should have low hysteresis, high linearity and high resolution.

8.

3. What is the difference between photoemmissive, photoconductive and photovoltaic transducers? Ans:
Photo emissive Transducer Photoconductive transducer Photovoltaic transducer

GRIET/ECE

www.jntuworld.com

www.jntuworld.com

www.jwjobs.net

Electronic Measurements & Instrumentation

Question & Answers

1. When cathode of photo emissive cell is subjected to radiations, it emits electrons. These electrons increase the current of the cell. Hence, this cell provides the measure of light intensity in terms of current. 2. It is a, passive transducer. 3. They have a moderate luminous sensitivity of 10100 uA/lm and 40-150 uA/lm. A very high sensitivity of 20 A/lm can also be achieved by photomultiplier tube. 4. The response time of these cells ' is less than that of photoconductive cells. 5. The spectral response of these cells range from visible to infrared region up to 220 nm.

When a photoconductive semiconductor element is subjected to radiations, its resistance changes (i.e., if decreases). Due to this, the flow of current through the cell increases.

When radiations fall on a photovoltaic cell, it generates a voltage whose value is proportional to the intensity of radiation incident on the cell. Thus, it converts electromagnetic energy into electrical energy.

This cell is a passive transducer. They are highly sensitive with small change in light intensity they exhibit high change in their resistance.

It is an active transducer. They possess sensitivity of 1 mA/lm.

These cells possess a short response time of 10-100 ps. These cells do not respond to radiations having wavelength less than 300 nm. Their spectral response extend from thermal radiation through visible, IR, UV up to X-rays and y-rays.

These cells have very short response time of 1-50 ps. The spectral response of these cells range from 200 nm - 2000 nm These cells are sensitive to, a-rays, (3-rays, y-rays and Xrays.

6. Photo emissive cells possess good amount of stability.

The characteristics of photoconductive cells are affected by temperature.

Photovoltaic cells are also stable but, they are seriously affected by temperature. An increase in temperature leads to a rapid decrease in output voltage of these cells, usually few mV/C.

GRIET/ECE

www.jntuworld.com

www.jntuworld.com

www.jwjobs.net

Electronic Measurements & Instrumentation

Question & Answers

7. They are used to measure luminous flux or luminous intensity.

These cells are widely used in measurement of radiant heat, quantitative spectroscopic measurements and pyrometry.

They are used for energy conversion purpose.

4. Explain working of semiconductor strain gauge and what are its specific advantages? Ans:
A typical semiconductor strain gauge is formed by the semiconductor technology i.e., the semiconducting wafers or filaments of length varying from 2 mm to 10 mm and thickness of 0.05 mm are bonded on suitable insulating substrates (for example Teflon). The gold leads are usually employed for making electrical contacts. The electrodes are formed by vapour deposition. The assembly is placed in a protective box as shown in the figure below.

elements used by the gauge are the materials such as GRIET/ECE

The strain sensitive semiconductor strain semiconductor silicon and 8

www.jntuworld.com

www.jntuworld.com

www.jwjobs.net

Electronic Measurements & Instrumentation

Question & Answers

germanium. When the strain is applied to the semiconductor element a large of change in resistance occur which can be measured with the help of a wheatstone bridge. The strain can be measured with high degree of accuracy due to relatively high change in resistance. A temperature compensated semiconductor strain gauge can be used to measure small strains of the order of 10-6 i.e., micro-strain. This type of gauge will have a gauge factor of 130 10% for a semiconductor material of dimension 1 x 0.5 x 0.005 inch having the resistance of 350 . Advantages of Semiconductor Strain Gauge 1. The gauge factor of semiconductor strain gauge is very high, about 130. 2. They are useful in measurement of very small strains of the order of 0.01 micro-strains due to their high gauge factor. 3. Semiconductor strain gauge exhibits very low hysteresis i.e., less than 0.05%. 4. The semiconductor strain gauge has much higher output, but it is as stable as a metallic strain gauge. 5. It possesses a high frequency response of 1012 Hz. 6. It has a large fatigue life i.e., 10 x 106 operations can be performed. 7. They can be manufactured in very small sizes, their lengths ranging from 0.7 to 7.0 mm.

5. What is temperature coefficient of resistor? Explain in detail? Ans:


The resistance thermometers (RTD) and thermistors employ the principle of change in electrical resistance with change in temperature. If the temperature changes, the resistance also changes due to changes in both length and sensitivity. Therefore materials used for resistance thermometers have temperature coefficient of resistivity much larger than the coefficient of thermal expansion. Thus, the temperature coefficient of resistance 'a' is given by

Where, T = Change in temperature, C /0 = Fractional change in resistivity R/R0 = Fractional change in resistance. 0 = Resistivity at 0 C R0 = Resistance at 0 C. The change in resistance with temperature can be given by the following relationship. R=R0(1+1T+2T2+-------------------+nTn+--------) Platinum, nickel, copper and tungsten are the commonly used resistance materials. These metals provide a definite resistance value at each temperature within its range. Curves indicate GRIET/ECE

www.jntuworld.com

www.jntuworld.com

www.jwjobs.net

Electronic Measurements & Instrumentation

Question & Answers

that the resistance of platinum and copper increases almost linearly with increasing temperature, while the characteristic of nickel is nonlinear. Tungsten has relatively high resistivity, but its use is limited for high temperature applications.

Gold and Silver are rarely used owing to their extremely low resistivity. Electrolytic copper which has highly consistent temperature coefficient of resistance i.e., higher than platinum. Due to their low relativity, their application is limited to low range industrial purposes. Phosphor bronze alloys are found suitable for low temperature measurements. Generally Nickel is chosen for resistance temperature measurements which has high temperature coefficient, less expensive than platinum and good reliability. According to their intended application the resistance material is selected.

6. Explain Piezo-electric effect? Ans:


When some pressure or stress is applied to the surface of the piezo-electric crystal, the dimensions of the crystal change and an electric charge voltage will be developed across certain surfaces of the piezo-electric crystal. Conversely, when an electric charge voltage or potential is applied to the crystal, the crystal get deformed and hence, the dimensions of it will change. This effect is referred as piezoelectric effect.

GRIET/ECE

10

www.jntuworld.com

www.jntuworld.com

www.jwjobs.net

Electronic Measurements & Instrumentation

Question & Answers

All the piezo-electric transducers work on the principle of piezo-electric effect. The piezoelectric materials or the materials used in the construction of piezoelectric crystal are quartz, rochelle salt, dipotassium titrate, lithium sulphate, barium titanate, potassium dihydrogen phosphate, ammonium dihydrogen phosphate etc. A piezoelectric crystal subjected to force is illustrated in figure 6.1.

7. Compare RTD with thermistor. Ans:


RTD 1. RTD is made up of metals. Thermistor Thermistor is made up of semiconductor materials Semiconductor materials have Negative Temperature Coefficient (NTC) of resistance. Hence, the resistance of a thermistor decreases with an increase in temperature and increases with a decrease in temperature. The resistance temperature characteristics of thermistor are highly nonlinear. It has large temperature coefficient of resistance i.e. It is highly sensitive to temperature. It has low operating temperature range compared to RTD i.e., -100 to + 300C.

2. Metals have Positive Temperature Coefficient (PTC) of resistance. Hence, the resistance of RTD increases with an increase in temperature and decreases with a decrease in temperature. 3. The resistance temperature characteristics of RTD's are linear. 4. It is less sensitive to temperature compared to thermistor. 5. But, it has-a wide operating temperature range i.e., - 200 to + 650C. 6. RTD's are relatively larger in size. 7. They are costlier. 8. They have low self resistance.

Thermistors are small in size. They are available at low costs. They have high self resistance. Thus, they require shielding cables to minimize interference problems. 11

GRIET/ECE

www.jntuworld.com

www.jntuworld.com

www.jwjobs.net

Electronic Measurements & Instrumentation

Question & Answers

9. RTD's provide high degree of accuracy and long term stability. 10. They are used in laboratory and industrial applications.

Thermistors also provide an accuracy of 0.01C. They are widely used for dynamic temperature measurement.

8. Explain briefly about poissons ratio? Ans:


Poisson's Ratio Poisson's ratio is defined as the ratio of lateral strain to axial strain. (or) It can also be stated as the ratio of unit strain in lateral (transverse) direction to the unit strain in axial direction. It is denoted by the symbol . =Lateral Strain/Axial Strain Derivation of Poisson's Ratio Consider, a steel bar of length ( L ) , and diameter D, as shown in the below figure.

When this steel bar is not subjected to any load, its dimensions (L and D ) remain the same. When a tensile load is applied to the bar, the bar habits a change in its dimensions. The tensile load act-the bar, increases the length of the bar in axial direction. Due to the increase in length, the diameter of e bar decreases and thus the cross sectional area of the bar changes i.e., the bar exhibits a change in its dimensions in lateral or transverse direction. Thus due to load, the bar experiences strain in axial direction as well as in lateral direction. Axial strain, a =Change in length/original length =L/L

GRIET/ECE

12

www.jntuworld.com

www.jntuworld.com

www.jwjobs.net

Electronic Measurements & Instrumentation

Question & Answers

Lateral strain, l = Change in diameter/original diameter = -D/D

9. Write short notes on resistive transducer? Ans:


Resistance of an electrical conductor is given by, R=l/A Where , R = Resistance in = Resistivity of the conductor ( - cm) l = Length of the conductor in cm. A = Cross-sectional area of the metal conductor in cm2 It is clear from the equation (1) that, the electrical resistance can be varied by varying, (i) Length (ii) Cross-sectional area and (iii) Resistivity or combination of these. Principle A change in resistance of a circuit due to the displacement of an object is the measure of displacement of that object Method of changing the resistance and the resulting devices are summarized in the following table. Method of changing resistance 1. Length - Resistance can be changed varying the length of the conductor, (linear and rotary). 2. Dimensions - When a metal conductor is subjected to mechanical strain, change in dimensions of the conductor occurs, that changes the GRIET/ECE

Resulting device Resistance potentiometers or sliding contact devices displacements

Use Used for the measurement of linear and angular.

Electrical resistance strain gauges.

Used for the measurement of mechanical strain.

13

www.jntuworld.com

www.jntuworld.com

www.jwjobs.net

Electronic Measurements & Instrumentation

Question & Answers

resistance of the conductor. 3. Resistivity - When a metal conductor is subjected to a change in temperature and change in resistivity occurs which changes resistance of the conductor.

Thermistor and RTD.

Used for the temperature measurement.

10. Draw the different forms of metal foil strain gauges and explain their principles of operation? Ans:
Metal Foil Strain Gauges In this type of strain gauges a metal foil is used to sense the applied strain. The materials used for its construction are nickel, nichrome, platinum, isoelastic (nickel + chromium + molybdenum), constantan (nickel + copper). The gauge factor and characteristics of foil strain gauges are similar to the wire strain gauges.

GRIET/ECE

14

www.jntuworld.com

www.jntuworld.com

www.jwjobs.net

Electronic Measurements & Instrumentation

Question & Answers

The metal foil gauges can be easily etched on a flexible insulating carrier film. In the construction of etched foil strain gauge first a layer of strain sensitive material is bonded to a thin sheet of backelite or paper. The part of some masking material and then to this unit an etching solution is applied. Therefore, the unmasked part of the metal will be removed thereby leaving the required grid structure. By this method of construction, the etched foil strain gauges are made in thinner sizes. Different forms of metal foil strain gauges are shown below. When a force or pressure is applied to the sensing element of metal foil strain gauge the physical dimensions of it will change. Since, the strain gauge element is pasted on its surface, the dimensions of the strain gauge changes due to which the resistance of the gauge changes. The measure of change in resistance will become the measure of applied pressure or force (this

GRIET/ECE

15

www.jntuworld.com

www.jntuworld.com

www.jwjobs.net

Electronic Measurements & Instrumentation

Question & Answers

change in resistance of the gauge can be measured by connecting the gauge in any one of the four arms of balanced Wheatstone bridge).

GRIET/ECE

16

www.jntuworld.com

www.jntuworld.com

www.jwjobs.net

Electronic Measurements & Instrumentation


Question & Answers

UNIT 8
1. Illustrate the principle of force summing devices using suitable examples and sketches? Ans: Force summing devices serve as primary transducers and convert the pressure applied at
the input into displacement, which then can be measured by means of secondary transducer. The lists of most widely used force summing devices are 1. 2. 3. Diaphragms Bellows Bourdon tubes

1. Diaphragms Any thin metal whose ends are fixed between two parallel plates is referred to as diaphragm. It is one of the pressure measuring elements. The operating principle is the applied pressure is converted into proportional displacement. The materials used to make diaphragms are phosphor bronze, nickel, beryllium copper, stainless steel, etc. These can be available in flat or corrugated shapes.

GRIET/ECE

www.jntuworld.com

www.jntuworld.com

www.jwjobs.net

Electronic Measurements & Instrumentation


Question & Answers

When two corrugated diaphragms are joined together at their ends a capsule is formed. When compared to flat diaphragms corrugated diaphragms produce greater displacements. Since the capsule is a combination of two diaphragms it generates more displacement which is twice that of the single corrugated diaphragm. This generated displacement is proportional to the applied pressure. 2. Bellows Bellows, the pressure measuring elements are formed by the series combination of capsules. The working principle of bellows is same as that of diaphragms i.e., the applied displacement is converted into proportionate mechanical displacement. The materials used to construct bellows are beryllium copper, brass, monel, stainless steel and nickel.

Whenever the pressure to be measured is applied the sealed end of bellow suffers displacement. The generated displacement can be known by attaching a pointer scale arrangement to the sealed end or by transmitting the displacement to the secondary transducer. 3. Bourdon Tubes The bourdon tubes are available in different shapes such as spiral, helical, twisted and C shaped. However all the tubes have non-circular cross-section. Also the materials used and working of all these types are same. The materials used in the construction of bourdon tubes are brass, steel and rubber.

GRIET/ECE

www.jntuworld.com

www.jntuworld.com

www.jwjobs.net

Electronic Measurements & Instrumentation


Question & Answers

The working principle of bourdon tube is same as that of diaphragms and bellows i.e., the applied pressure is converted into mechanical displacement. The displacement generated by the above force summing devices can be converted into electrical form by transmitting it to LVDT. The output voltage generated by LVDT is proportional to displacement and hence applied pressure.

2. Explain general Data Acquisition System (DAS) with a neat block diagram? Ans:
The block diagram of a general Data Acquisition System (DAS) is shown in the figure below. It consists of the following elements. 1 . Transducer 2 . Signal conditioner 3 . Multiplexer 4 . Analog to Digital Converter 5 . Recorders and Display devices

GRIET/ECE

www.jntuworld.com

www.jntuworld.com

www.jwjobs.net

Electronic Measurements & Instrumentation


Question & Answers

1. Transducer A transducer is used to convert the physical parameters corning from the field into electrical signals or it is used to measure directly the electrical quantities such as resistance, voltage, frequency, etc. 2. Signal Conditioner Usually the output signals of the transducer will be of very low level (weak) signals which cannot be used for further processing. In order to make the signals strong enough to drive the other elements signal conditioners such as amplifiers, modifiers, filters etc., are used. 3. Multiplexer The function of the multiplexer is to accept multiple analog inputs (after signal conditioning) and provide a single output sequentially according to the requirements. 4. A/D Converter The analog-to-digital (A/D) converter is generally used to convert the analog data into digital form. The digital data is used for the purpose of easy processing, transmission, digital display and storage. Processing involves various operations on data such as comparison, mathematical manipulations, data is collected, converted into useful form and utilized for various purposes like for control operation and display etc. The transmission of data in digital form is possible over short distances as well as long distances of and has advantages over transmission in analog form. The data can be stored permanently or temporarily and can be displayed on a CRT or digital panel. 5. Recorders and Display Devices In display devices the data is displayed in a suitable form in order to monitor the input signals. Examples of display devices are oscilloscopes, numerical displays, panel meters, etc. In order to have either a temporary or permanent record of the useful data recorders are used. The analog data can be recorded either graphically or on a magnetic tape. Optical recorders, ultraviolet recorders, styles-and-ink recorders are some of its examples. The digital data can be recorded through digital recorders. The digital data is first converted into a suitable form for recording by means of a coupling unit and then recorded on a magnetic tape, punched cards or a perforated paper tape.

GRIET/ECE

www.jntuworld.com

www.jntuworld.com

www.jwjobs.net

Electronic Measurements & Instrumentation


Question & Answers

3. Show and explain the capacitive transducer arrangement to measure angular velocity and what are its limitations? Ans:
The arrangement of capacitive transducer in the arrangement of angular velocity is shown figure 3.1

The main components of a capacitive tachometer arrangement are given as follows, 1. Fixed capacitor plates 2. A vane attached to one of the two ends of a shaft 3. A pulse shaper and amplifier circuit 4. An electronic counter or frequency meter. The vane is placed between the two fixed plates of capacitor and the free end of the shaft is connected to the source whose angular velocity is to be determined. Therefore the shaft rotates along with the source, which in turn rotates the vane between the plates. Due to this the capacitance of the capacitor changes. For every rotation of the vane a change in capacitance takes place and for every changed capacitance value, a voltage pulse is induced. The number of times the capacitance value changes per unit time gives the angular velocity of the rotating shaft. The induced pulses are applied to pulse shaper and amplifier circuit which shapes the pulses into accurate pulses and then amplifies the pulses. These shaped and amplified pulses are then applied to electronic counter which counts the number of pulses. The counted number of pulses directly gives the value of angular velocity. Limitations: 1. Capacitive transducers are highly sensitive to temperature. Therefore any variation in temperature affects the performance of the instrument. 2. High output impedance of capacitive transducers lead to loading effects. GRIET/ECE

www.jntuworld.com

www.jntuworld.com

www.jwjobs.net

Electronic Measurements & Instrumentation


Question & Answers

3. The presence of duct particles, moisture, etc., changes the capacitance of the capacitor. Due to this error occurs in the output.

4. What are the main elements of velocity transducer? The main elements of a velocity transducer are coil and a permanent magnet. In Ans:
such type of transducers velocity is measured based upon electromagnetic induction principle. These two elements can be arranged in two different configurations (i.e., electrodynamic and electromagnetic) to measure the velocity. In electrodynamics velocity transducer, moving coil scheme is employed. In this configuration the coil and the magnet are arranged in housing such that the magnet is attached to the base of the housing and the coil is attached to the other side (top) of the housing with the help of a spring so that the coil is suspended in the magnetic field as shown in the figure below. The body whose velocity is too measured is connected to the base of the housing. Due to the displacement of the body, the housing also gets displaced which in turn caused a displacement of the coil in the magnetic field. This movement of the coil causes a change in the flux linkages between the Magnet and the coil, and thus according to the electromagnetic induction principle an electrical voltage gets induced in the coil. This induced voltage is proportional to the relative velocity of the spring and is given by the equation. e = BLV r X 10-8 Where, e-Induced voltage B-Flux density L-Length of coil Vr=dx/dt=relative velocity of coil with respect to magnet (cm/s)

GRIET/ECE

www.jntuworld.com

www.jntuworld.com

www.jwjobs.net

Electronic Measurements & Instrumentation


Question & Answers

5). Explain about Linear Variable Diffrential Transformer (LVDT)? Ans: Linear Variable Differential Transformer (LVDT) consists of one primary winding (P)
and two secondary windings (51 and S2).with equal number of turns wound on a cylindrical former. The two secondary windings are connected in series opposition and are placed identically on either side of primary winding to which an AC excitation voltage is connected. A movable soft iron core is placed within the cylindrical former. When the displacement to be measured is applied to the arm of the core, the LVDT converts this displacement into an

electrical signal. The construction of LVDT is illustrated in figure (5.1).

The operating principle of LVDT depends on mutual inductance. When the primary winding is supplied with A.C. supply voltage, it generates alternating magnetic field. Due to this magnetic field an alternating voltage will be induced in the two secondary windings. In the figure (5.2) es1 is the output voltage of secondary winding S1 and es2 is the output voltage of secondary winding S2 In order to get single differential output voltage two secondary windings are connected in series opposition. Thus the differential output voltage is given by, e0=es1-es2 When the core is placed symmetrically with respect to two secondary windings an equal amount of voltage will be induced in both windings. Therefore esl - es2 and the output voltage is '0'. Hence, this position is known as null position. Now if the core is moved towards up from null position, more magnetic field links with secondary winding S1, and small field links with secondary winding S2. Therefore more voltage will be induced in S 1 a n d l e s s i n S 2 i . e . , e s 1 will be larger than e s 2 . Hence the differential output voltage is e0=es1-es2 and is in phase with primary voltage.

GRIET/ECE

www.jntuworld.com

www.jntuworld.com

www.jwjobs.net

Electronic Measurements & Instrumentation


Question & Answers

But when the core is moved towards down from null position more magnetic field links with secondary winding S2 and small field links with secondary winding S1. Therefore more voltage will be induced in S2 and less in S1, i.e.,es2 will be larger than es1. Hence, the differential output voltage is e0 = es2 es1 and is 180 out of phase with primary voltage. Thus the output voltage e0 position of the core and hence the displacement applied to the arm of the core. Merits 1. LVDT has good linearity i.e.. it produces linear output voltages. 2. It can measure displacements of very high range usually from 1.25mm to 250mm. 3. It has high sensitivity. 4. Since it produces high output, it does not require amplifier devices. 5. It has low hysteresis. 6. It consume less power (about < 1w) Demerits 1. It is sensitive to stray magnetic fields. 2. Performance of LVDT is affected by variations in temperature. 3. It has limited dynamic response. 4. To provide high differential output, it requires large displacements.

6. Explain spiral type bourdon tube ? Ans:


Principle is deflection generated give the pressure. the bourdon and Working The working principle of spiral type bourdon tube is that the applied pressure converted into mechanical or displacement. The displacement will value of applied The materials used in construction of tube are brass, steel rubber. Construction and

GRIET/ECE

www.jntuworld.com

www.jntuworld.com

www.jwjobs.net

Electronic Measurements & Instrumentation


Question & Answers

Spiral type bourdon tubes are constructed by winding the long tube into several turns in the form spiral shape. One end of the bourdon tube is opened through which the pressure to be measured is applied whereas another end is closed. The sealed end of the tube is connected to a pointer mechanically. Whenever the fluid whose pressure is to be known is applied to the open end of the spiral tube, it tends to uncoil. Due to this a long movement of the tip (end) takes place and this displacement is transmitted to pointer. Therefore the pointer moves on the calibrated scale, thereby indicating the applied pressure. When compared to C-type bourdon tube spiral type bourdon tube produces the results with very high accuracy.

7. Explain how pressure is measured using Piezoelectric transducer? Ans:


Principle and Design Piezoelectric pressure transducers depend on the principle of piezoelectric effect i.e., when some pressure or stress is applied to the surface of the piezoelectric crystal, an electric charge voltage will be developed by the crystal. The materials used in the construction of piezoelectric crystals are quartz, Rochelle salt, dipotassium titrate, lithium sulphate, barium titanate etc. A piezoelectric pressure transducer is formed by connecting a diaphragm to the piezoelectric crystals and this assembly is shown below.

GRIET/ECE

www.jntuworld.com

www.jntuworld.com

www.jwjobs.net

Electronic Measurements & Instrumentation


Question & Answers

Working The pressure which is to be measured is applied to corrugated metal diaphragm. The diaphragm deflects depending on the applied pressure, and this deflection signal is transmitted to the crystal through the mechanical link. In other words, the pressure is applied to the crystal through the diaphragm and the link. When the crystal senses the pressure it will generate some voltage corresponding to the applied pressure, and is measured in the output voltage measuring device which is calibrated in terms of applied pressure. Applications (a) These used in process can be the which

GRIET/ECE

10

www.jntuworld.com

www.jntuworld.com

www.jwjobs.net

Electronic Measurements & Instrumentation


Question & Answers requires measurement of pressure. applied in those systems which requires measured variable in electrical form.

high (b) Can be

Merits 1. Provides electrical output. 2. This transducer does not require any external power supply. 3. Size in small. 4. Rugged construction. Demerits 1. It cannot be used for static pressure measurements. 2. The response will get affected by the variations in temperature. 3. In some cases it requires signal conditioning circuitry which is complex. 4. Cost is high. 8. Briefly explain the working principles and measurement of force by any two nonelectric techniques? Ans: The principles used for the measurement 'of force by non-electrical techniques are, 1. 2. Balance principle Force to pressure conversion principle.

In balance principle, the force to be measured is balanced by a known certain quantity and then the value of unknown force is determined from the amount of balancing quantity required to balance the force. In force to pressure conversion principle, the unknown force is converted into pressure. This pressure is measured and then the value of unknown force is determined from the measure of this pressure. Based on the above principles, the nonelectrical devices used for measurement of force are as follows, (i) Balance type force measuring device (ii) Hydraulic and pneumatic load cell.

(i) Balance type force measuring device: The force measuring device based upon the principle of balance is usually a simple lever system as shown in figure (8.2) below. This system consists of a rod resting on a pivot at a certain point along its length.

GRIET/ECE

11

www.jntuworld.com

www.jntuworld.com

www.jwjobs.net

Electronic Measurements & Instrumentation


Question & Answers

The force to be measured is applied on one of the rod, at a distance T from the pivot. This makes the lever system to be unbalanced. To balance the system, a mass of known value m' is placed on the other end of the rod, at a distance T from the pivot. At balance, the lever system satisfies the following condition.
FL = mgl

Where, g = Acceleration due to gravity/gravitational constant. The unknown force F can be calculated from the above balance equation.

(ii)Hydraulic and Pneumatic Load Cells The hydraulic and pneumatic load cells are non-electric type force measuring devices, which provide the measure of force in terms of pressure. In a hydraulic load cell, a chamber is filled with a liquid (usually oil) and the top/mouth of the chamber is filled with a diaphragm (i.e., the internal edge of the diaphragm is in contact with the oil). The force to be measured is applied on top of 'the diaphragm.

Due

to the application 12

GRIET/ECE

www.jntuworld.com

www.jntuworld.com

www.jwjobs.net

Electronic Measurements & Instrumentation


Question & Answers

of force, the diaphragm gets deflected and downwards, thereby produces pressure on the liquid.

The pressure indicator connected to the chamber indicates the amount of liquid pressure created by the force. Very large forces of the order of mega Newtons can be measured by a hydraulic cell. The pneumatic load cell consists of a diaphragm at one end of its chamber and a nozzle at the other end as shown in the figure below. The chamber is provided with a supply of air, under pressure. When unknown, force is applied on the diaphragm, the diaphragm deflects. Due to deflection of the diaphragm, the gap between the nozzle and the diaphragm changes. As a result the air pressure in the chamber also changes. As the gap reduces, the pressure in the chamber increases. A pressure indicator attached to the chamber indicates the air pressure in the chamber. The unknown force can then be evaluated from the measure of the air pressure. Forces upto 20KN can be measured by pneumatic load cell.

GRIET/ECE

13

www.jntuworld.com

www.jntuworld.com

www.jwjobs.net

Electronic Measurements & Instrumentation


Question & Answers

9). Explain the working principle of potentiometric type accelerometer? Ans A potentiometric accelerometer employs a seismic mass, spring arrangement, dashpot,
and a resistive element. The seismic mass (potentiometer) is connected between spring and dashpot. The wiper of the potentiometer is connected to the mass.

In the presence of vibration or acceleration, vibrational displacement of seismic mass takes place with respect to the housing of the device. The displacement of mass is transferred to the potentiometers through the wiper. Therefore the resistance of the potentiometer changes. This change in resistance gives the value of displacement and hence the acceleration. Advantages 1. Construction and operation are very simple. 2. Low cost. Disadvantages 1. Resolution is low. 2. They cannot be suitable for high frequency vibrations.

10. Explain the stroboscopic method of measuring the angular speed? Ans:
Stroboscopic Method The periodic or rotary motions can be measured by using a device known as stroboscope. A stroboscope is a device that consists of a source of variable frequency flashing GRIET/ECE

14

www.jntuworld.com

www.jntuworld.com

www.jwjobs.net

Electronic Measurements & Instrumentation


Question & Answers

brilliant light called Strobotron. The flashing frequency of Strobotron is controlled by a variable frequency oscillator.

The principle involved in measurement of speed through stroboscope is to make the moving objects visible only at specific intervals of time by adjusting the flashing frequency. The figure below shows a stroboscope measuring the speed of shaft. The speed of the shaft using a stroboscope is measured in the following manner. An identification mark is made directly on the shaft or on a disc mounted on the shaft. The flashing light from the stroboscope is made to fall on the mark and the frequency of flashing is adjusted so that the mark appears to be stationary. Under such condition the speed of rotation is equal to the flashing frequency. The speed can be read directly from the scale of the stroboscope which is calibrated in terms of speed.

GRIET/ECE

15

www.jntuworld.com

You might also like